Pain Managment

179
Pain Managment

description

Pain Managment. - PowerPoint PPT Presentation

Transcript of Pain Managment

Page 1: Pain Managment

Pain Managment

Page 2: Pain Managment

A 40-year-old male with chronic hepatitis C has osteoarthritis in his knees that is beginning to limit

his activity. He asks you if he can take acetaminophen for the pain.

Which of the following would be appropriate advice? (Mark all that are true.)Acetaminophen

overdose is a leading cause of fulminant liver failure in adultsAcetaminophen is excreted through the

biliary systemHe can safely take up to 3 grams of acetaminophen per dayNSAIDs are preferred over

acetaminophen in patients with chronic liver disease

Page 3: Pain Managment

Answer

• Acetaminophen overdose is a leading cause of fulminant liver failure in adults

Page 4: Pain Managment

Acute acetaminophen overdose is a very common problem in the United States, and when

unrecognized can lead to fulminant hepatic failure. In healthy adult nondrinkers, acetaminophen is safe taken chronically in doses up to 4 g/day. Adults who drink excessively, those with chronic liver disease, and those with malnutrition are at increased risk for

toxicity. Acetaminophen should be limited to 2 g/day in these persons (B level recommendation). It

appears safe at this dosage, and is preferred over NSAIDs in patients with chronic liver disease. Acetaminophen is metabolized in the liver and

excreted by the kidneys.

Page 5: Pain Managment

A 40-year-old male has had low back pain for 2 years. He asks your advice concerning physical

therapy.

Which of the following would be appropriate advice? (Mark all that are true.)

Prescribed exercise programs are the most efficacious physical modality for chronic back pain

Transcutaneous Electrical Nerve Stimulation (TENS) units produce modest benefits in pain

reductionRegular massage therapy often produces lasting

benefitsWhile expensive, multidisciplinary rehabilitation

programs are clearly beneficialHydrotherapy is ineffective for chronic back pain

Page 6: Pain Managment

Answer

• Prescribed exercise programs are the most efficacious physical modality for chronic back painWhile expensive, multidisciplinary rehabilitation programs are clearly beneficialHydrotherapy is ineffective for chronic back pain

Page 7: Pain Managment

It is difficult to analyze the evidence for the efficacy of physical therapy, because improvement is affected by a patient's effort and motivation, as

well as the personal attention one gets from the physical therapist. Randomized, controlled trials are difficult to perform and compare. On this subject, systematic reviews and meta-analyses do not always agree.

A review of physical modalities for chronic back pain published in 2004 looked not only at efficacy, but also at the clinical significance of the effect. Only exercise programs and multidisciplinary rehabilitation programs (which can cost thousands of dollars) were shown to be

effective and clinically beneficial. Laser therapy, spinal manipulation, and massage were shown to be mildly effective with little lasting clinical

benefit. Using the same criteria, TENS, magnets, ultrasound, hydrotherapy, and traction were ineffective. There was too little evidence

to rank acupuncture, back schools, and lumbar supports.

Physical therapists not only administer modalities but also provide functional assessments, patient evaluations, and patient education. Therapists can specialize in areas such as neurologic rehabilitation,

wound management, or sports training.

Page 8: Pain Managment

Two weeks ago, a 30-year-old female with a history of lymphoma underwent her sixth and last cycle of chemotherapy before radiologic reevaluation. She has had chronic lymphoma-related back pain for the past six months, adequately controlled by oxycodone/acetaminophen (Percocet), 5 mg/325 mg, one to two tablets orally every 4 hours as

needed. During her last chemotherapy cycle she became neutropenic, and was treated with filgrastim (Neupogen).

The patient comes to your office today for regular follow-up of prednisone-induced hyperglycemia, and complains of severe bilateral

lower extremity pain. The pain started about 2 weeks ago and is mostly over her shins. She tells you it is a constant, sharp pain, and that the

Percocet is not relieving her pain anymore. 

What is the most likely cause of her new pain? Pain secondary to increased cytokines from chemotherapy-related

tumorlysis 

Neuropathic pain related to her prednisone-induced hyperglycemia 

Osteoporosis-related pain from high-dose prednisone 

Bone pain from increased bone marrow activity resulting from treatment with filgrastim 

Tolerance to Percocet secondary to chronic use

Page 9: Pain Managment

Answer

• Bone pain from increased bone marrow activity resulting from treatment with filgrastim

Page 10: Pain Managment

Filgrastim is used to treat neutropenia from chemotherapy or bone marrow transplantation. It stimulates granulocyte and

macrophage proliferation and differentiation. One of its most common side effects is bone pain (30% of patients) that

usually starts in the first 3 days of treatment.

Although tumor lysis can cause pain from increased circulating cytokines, this pain is usually diffuse. Long-lived

hyperglycemia does cause neuropathic pain that is usually described as burning, shooting, "pins and needles," and

"painful numbness." The onset is very rarely acute, however. Osteoporosis does not cause pain. 

Tolerance does not occur abruptly. Any pain escalation should be thoroughly investigated before a diagnosis of tolerance is made, especially when the pain has an acute

onset.

Page 11: Pain Managment

You decide to use a pain rating scale to help guide the management of a 77-year-old

female with peripheral neuropathy who has daily pain. 

True statements regarding these scales include which of the following? (Mark all that are

true.)They are simple and easy to administerThey eliminate embellishment of painThe emotional state of the patient can influence the ratingThere are valid scales

which are useful in children

Page 12: Pain Managment

Answer

• They are simple and easy to administer

• The emotional state of the patient can influence the rating

• There are valid scales which are useful in children

Page 13: Pain Managment

Pain rating scales are used by many physicians to measure intensity of pain and monitor the effect of therapy. (C level recommendation,

evidence level II) They can be administered in a matter of minutes and are easy to score. These scales can be adapted for any age group by

substituting words and/or pictures. Most children understand that 10 is greater than 2 and can use the simple 1-10 scale.

There are four scales in wide use: numeric rating scales, verbal rating scales, visual analog scales, and pain drawings. Some clinicians use pain diaries to further enhance the rating's accuracy and provide information

on function. One drawback is that the scales allow for embellishment and can be skewed one direction or the other by someone with little pain

experience (Evidence level III). A teenager who is naive to pain might rate the pain of a sore throat as a 10, or a patient seeking sympathy or

additional treatment might overrate the amount of pain he or she is having. Emotions affect the pain experience and can also affect the rating.

While there is some controversy about how accurate these scales are, most clinicians have learned to use them for assessment and management

of pain.

Page 14: Pain Managment

40-year-old nurse presents with neck pain. He has no history of specific injury. His symptoms are

intermittent and seem worse when turning his head to the left. The pain radiates into the left thumb and

index finger. At times it can be severe.

Which one of the following is true regarding this problem?

 The irritated nerve root is most likely C5 Diagnostic imaging would be helpful Flexion of the neck is

likely to worsen the pain The most common cause is osteophytes

Page 15: Pain Managment

Answer

• Diagnostic imaging would be helpful

Page 16: Pain Managment

Cervical radiculitis is quite common and results from irritation of the cervical nerve as it leaves the spinal cord. The

location of the symptoms may help to identify the irritated nerve. A C5 root irritation will cause pain in the shoulder but

without radiation into the arm. The pain is generally made worse by extension, a maneuver that decreases the space for

the nerve root. Flexion may actually help the pain. The pain is generally also made worse by turning toward the side of the

compression. Reflexes decrease as the duration of the compression lengthens.

The most common cause of cervical nerve root compression is either an acute disc herniation or a degenerative disc.

Osteophytes may cause no impingement. Diagnostic imaging is helpful and should start with plain films of the cervical

spine with oblique views (C level recommendation). An MRI is the most definitive imaging test, but should be used to

confirm the diagnosis since there is a fair amount of asymptomatic cervical pathology (Evidence level II).

Page 17: Pain Managment

Which one of the following agents provides the greatest

analgesic effect when compared milligram to milligram? Oxycodon(OxyContin) 

Morphine(MSContin) 

Codeine 

Hydromorphone(Dilaudid) 

Hydrocodone

Page 18: Pain Managment

Answer

• Hydromorphone (Dilaudid)

Page 19: Pain Managment

True statements regarding the management of vertebral compression fractures in the elderly include

which of the following? (Mark all that are true.)These fractures are normally unstable, making

surgical treatment idealIf conservative treatment is selected, a minimum of 4

weeks of bed rest is requiredIn the elderly, NSAIDs are safer than opioids

Calcitonin-salmon (Miacalcin) nasal spray can be used for treatment of pain

Percutaneous vertebroplasty can be helpful in managing the pain when conservative treatment fails

After the fracture heals, returning to a normal exercise program may still be dangerous

Page 20: Pain Managment

Answer

• Calcitonin-salmon (Miacalcin) nasal spray can be used for treatment of painPercutaneous vertebroplasty can be helpful in managing the pain when conservative treatment fails

Page 21: Pain Managment

Compression fractures of the vertebral body are common, especially in older adults. Vertebral compression fractures usually are caused by osteoporosis, and range from mild to severe. More severe fractures can cause significant pain, leading to the inability to perform activities of daily living, and life-threatening decline in the elderly patient who already has decreased

reserves. While the diagnosis can be suspected from the history and physical examination, plain radiographs are often helpful for determining the diagnosis and prognosis. Occasionally, it may also be helpful to obtain CT or MRI.

The physician must first determine if the fracture is stable or unstable. A stable fracture will not be displaced by physiologic forces or movement. Fortunately, compression fractures are normally stable as a result of their impacted nature. Traditional treatment is non-operative and conservative. Patients are treated with a short period (no more than a few days) of bed rest.

Prolonged inactivity should be avoided, especially in elderly patients. Oral or parenteral analgesics may be administered for pain control, with careful observation of bowel motility. If bowel sounds and flatus are not present, the patient may require

evaluation and treatment for ileus. Calcitonin-salmon nasal spray can be used for treatment of pain. Muscle relaxants, external back braces, and physical therapy modalities also may help (Evidence level B). NSAIDs have been shown to

significantly increase gastrointestinal bleeding in the elderly and must be used with caution (Evidence level A, randomized controlled trial).

Patients who do not respond to conservative treatment or who continue to have severe pain may be candidates for percutaneous vertebroplasty. This involves injecting acrylic cement into the collapsed vertebra to stabilize and strengthen the fracture and vertebral body. This procedure does not, however, restore the shape or height of the compressed vertebra. Kyphoplasty, where cement is injected into a cavity created by a high-pressure balloon, is being evaluated for use and may

be successful in restoring height to the collapsed vertebra.

Most patients can make a full recovery or at least significant improvements within 6-12 weeks, and can return to a normal exercise program after the fracture has fully healed. A well-balanced diet, regular exercise program, calcium and vitamin D

supplements, smoking cessation, and medications to treat osteoporosis (such as bisphosphonates) may help prevent additional compression fractures. Age should never preclude treatment.

There is now good evidence that diagnosing and treating osteoporosis does indeed reduce the incidence of compression fractures of the spine (Evidence level A). Regular activity and muscle strengthening exercises have been shown to decrease

vertebral fractures and back pain. Measures to prevent falls must be initiated by patients and their caregivers.

Family physicians can help patients prevent compression fractures by diagnosing and treating predisposing factors, identifying high-risk patients, and educating patients and the public about measures to prevent falls.

Page 22: Pain Managment

You are seeing a colleague's patient for a follow-up visit. The patient has been taking opioids for 12

months for chronic low back pain. After reviewing his chart you notice numerous phone messages from

the patient asking for early refills, as he has been using his opioids more frequently than prescribed.

During the encounter, the patient admits to borrowing pain medications from his wife.

In the absence of other aberrant behavior, this is highly indicative of

 dependence 

addiction 

pseudoaddiction 

drug trafficking

Page 23: Pain Managment

Answer

• pseudoaddiction

Page 24: Pain Managment

The suspicion of opioid addiction in chronic pain sufferers is often triggered by the occurrence of what have been called aberrant drug-related behaviors. Ambiguities inherent in this approach affect patient care adversely. Rather than consistently

signifying abuse or addiction, these behaviors are often motivated by undertreated pain.

The term pseudoaddiction was coined in 1989 to describe chronic pain victims mistakenly diagnosed as suffering from opioid addiction when undertreated pain led to certain drug-related behaviors. Simply stated, pseudoaddiction is a

misdiagnosis that results from undertreatment of chronic pain. Patients are frequently harmed by the misdiagnosis of addiction and these behaviors should prompt an aggressive search for undertreatment of pain. Unfortunately, this usually does not happen. Instead, when a patient displays certain behaviors, he or she is typically threatened with termination of

treatment, rather than questioned about its effectiveness.

Undertreatment of chronic pain should be considered first on the list of differential diagnoses when considering the cause of worrisome drug-related behaviors. Some of these behaviors include

borrowing another patient's drugsobtaining prescription drugs from nonmedical sources

unsanctioned dosage escalationsaggressive complaining about the need for higher doses

drug hoarding during periods of reduced symptomsrequesting specific drugs

acquisition of similar drugs from medical resources

(Evidence level III)The diagnosis of opioid addiction should be based on observation of deteriorating function, which can be directly attributed to opioid abuse, rather than inferred from an anecdotal set of behavioral criteria derived from medical folklore. Behaviors

suggestive of opioid addiction include injection of substances prescribed for oral use, concurrent use of related illegal drugs, and selling prescription drugs (Evidence level III).

When patients are obtaining opioids from more than one medical source, the primary physician must reevaluate the pain syndrome. Factors to consider include whether the patient is undermedicated, whether the syndrome is misdiagnosed, and

whether the patient is abusing or diverting drugs.

Physicians must work with chronic pain patients to adequately evaluate and treat their pain. In turn, physicians must expect patients to use only one source to obtain opioid prescriptions. A written opioid use agreement is recommended.

Page 25: Pain Managment

Mind-body therapy (MBT), such as relaxation, (cognitive) behavioral therapies, meditation, imagery, biofeedback, and

hypnosis, is used for several common clinical conditions. There is good evidence to support which of the following

statements about MBT? (Mark all that are true.)MBT is more effective for decreasing pain intensity than for improving functional status associated with low back pain

MBT has NO significant effect in the symptomatic treatment of arthritis

Stress management training can be as effective as tricyclic antidepressants in the management of chronic tension-type

headacheThe combination of relaxation training and thermal

biofeedback is the preferred behavioral treatment for recurrent migraine disorder

Page 26: Pain Managment

Answer• MBT is more effective for decreasing pain

intensity than for improving functional status associated with low back pain Stress management training can be as effective as tricyclic antidepressants in the management of chronic tension-type headacheThe combination of relaxation training and thermal biofeedback is the preferred behavioral treatment for recurrent migraine disorder

Page 27: Pain Managment

Multimodal mind-body therapy (MBT) treatments typically include some combination of relaxation, biofeedback therapy, cognitive strategies (e.g., for coping with pain), and

education. Narrative reviews suggest that the Arthritis Self-Management Program (ASMP) might be a particularly effective adjunct in the management of arthritis (Evidence level III). This community-based program consists of education, cognitive restructuring, relaxation, and physical activity to reduce pain and distress and facilitate problem solving. Using this program, reductions in pain were maintained 4 years after the intervention, and physician

visits were reduced by 40% (Evidence level II).

A review of the efficacy of MBTs in chronic low back pain concluded that there was strong evidence (defined as generally consistent findings in multiple high-quality

randomized, controlled trials) that MBTs, when compared with wait-list controls or usual medical care, have a moderate positive effect on pain intensity and only small effects on

functional status and behavioral outcomes (Evidence level I, Cochrane review).

A review of the efficacy of relaxation and biofeedback in recurrent migraine headache showed a 43% reduction in headache activity in the average patient compared with a 14% reduction with placebo medication and no reduction in unmedicated subjects (Evidence

level II). A more recent narrative review concluded that a combination of relaxation training and thermal biofeedback is the preferred behavioral treatment for recurrent migraine disorder (C level recommendation). Recent evidence indicates that stress

management training is as effective as tricyclic antidepressants in the management of chronic tension-type headache, suggesting that combining these two therapeutic

approaches might be more effective than using either one alone (Evidence level I).

Page 28: Pain Managment

True statements regarding dysmenorrhea include which of the following? (Mark

all that are true.)Leiomyomata can cause secondary

dysmenorrheaOral contraceptives will not help primary

dysmenorrheaNSAIDs can be used on an intermittent

basis to help with dysmenorrheaProstaglandins play a principal role in

dysmenorrhea

Page 29: Pain Managment

Answer

• Leiomyomata can cause secondary dysmenorrheaNSAIDs can be used on an intermittent basis to help with dysmenorrheaProstaglandins play a principal role in dysmenorrhea

Page 30: Pain Managment

Dysmenorrhea is pain that occurs during the menses and is crampy in nature. It is commonly classified as either primary or secondary. Primary dysmenorrhea is a condition unto itself

that is not a symptom of another disorder. Secondary dysmenorrhea can be caused by leiomyomata or by other pelvic pathology. Prostaglandin release is the understood

pathophysiology for primary dysmenorrhea. Oral contraceptives provide relief for primary dysmenorrhea by suppressing ovulation and thereby reducing the release of prostaglandins (Evidence level II). NSAIDs that inhibit

prostaglandin synthetase provide relief in most patients and are usually initiated for 2-5 days, just before and during the

menses (Evidence level I). In some recalcitrant cases the NSAIDs could be used continuously, with proper attention to

the risks of chronic NSAID use.

Page 31: Pain Managment

A 34-year-old female presents with intermittent facial pain. The pain occurs in brief episodes and always on the left side of her face. She reports that the pain is like an electric shock. The episodes may be evoked by smoking, talking, or washing her face. There are times, however, where there is no inciting stimulus. Between episodes she is pain free and there are no

sensation deficits on her face.

True statements regarding this problem include which of the following? (Mark all that are true.)

Facial sensory loss associated with facial pain should prompt cerebral imaging

This may be the first manifestation of multiple sclerosisA large proportion of cases are caused by compression of the

nerve by a blood vesselCarbamazepine (Tegretol) is first-line medical management

Patients not responding promptly to pharmacotherapy should be offered referral for interventional therapy

Page 32: Pain Managment

Answer• Facial sensory loss associated with facial pain should

prompt cerebral imagingThis may be the first manifestation of multiple sclerosisA large proportion of cases are caused by compression of the nerve by a blood vesselCarbamazepine (Tegretol) is first-line medical managementPatients not responding promptly to pharmacotherapy should be offered referral for interventional therapy

Page 33: Pain Managment

Trigeminal neuralgia (TGN) is a painful condition which affects one side of the face. It is characterized by brief shock-like pain limited to the distribution of one or more divisions of

the trigeminal nerve. The pain may be stimulated by such actions as washing, shaving, smoking, talking, or brushing the teeth, but may also occur spontaneously. It begins and ends

abruptly, and may remit for varying periods.

Loss of facial sensation or any suspected involvement of a cranial nerve should prompt appropriate cerebral imaging (C level recommendation). In the last three decades, evidence has been mounting that in a large proportion of cases, compression of the trigeminal nerve

root at or near the dorsal root entry zone by a blood vessel is a major causative or contributing factor (Evidence level III). Of the known etiologic factors, the association of

multiple sclerosis (MS) with TGN is well established. MS is seen in 2%-3% of patients with TGN. Conversely, TGN is diagnosed in 1%-5% of patients with MS. In a small proportion of

patients with MS, TGN is the first manifestation of the disease.

Pharmacotherapy remains the mainstay of treatment of TGN. Unfortunately, only a few randomized, controlled trials have been conducted. Carbamazepine (Evidence level I),

oxcarbazepine (Evidence level II), phenytoin (Evidence level III), lamotrigine (Evidence level II), and baclofen are commonly used to treat TGN. Patients with TGN are often willing

to consider surgery as a first-line treatment in anticipation of a permanent cure. Numerous interventional procedures (e.g., cryotherapy, alcohol blocks, radiofrequency lesions) and

operations (e.g., microvascular decompression) are available to treat TGN. Each is associated with complications and recurrences. Patients should be provided a realistic view and

balanced information regarding treatment choices.

Page 34: Pain Managment

A 52-year-old male is admitted to the hospital with abdominal pain and dehydration, and is diagnosed with

inoperable pancreatic cancer. He chooses to return home with hospice care. While in the hospital, he has been using patient-controlled analgesia (PCA). His PCA is set to deliver 1 mg of intravenous morphine on demand, with a 10-minute lockout. There is no basal rate. He reports his pain is well controlled. Over the past 3 days, his morphine use has been 28 mg/day,

32 mg/day, and 29 mg/day.

You wish to send the patient home on sustained-action opioids to help control his pain. Based on his PCA usage,

which one of the following would be an appropriate starting dosage of a sustained-action morphine (MS Contin)?

 15 mg orally twice daily 30 mg orally twice daily 

45 mg orally twice daily 60 mg orally twice daily

Page 35: Pain Managment

Answer

• 45 mg orally twice daily

Page 36: Pain Managment

While there is some variability, 10 mg of parenterally administered morphine is

approximately equivalent to 30 mg orally (Evidence level I). On average, this patient

has used 29.7 mg of intravenous morphine per day. This would be approximately equivalent

to 90 mg of oral morphine per day. An appropriate dosage of sustained-action oral

morphine would be 45 mg twice daily, or 30 mg three times daily. Milligram-to-milligram, oxycodone is about 50% more powerful than

morphine.

Page 37: Pain Managment

Important concepts for assessing pain in older adults with cognitive impairment include which of the

following? (Mark all that are true.)Observing for changes in normal functioningAsking about pain using synonyms, such as

discomfort, aching, and sorenessFraming questions in the present tense (e.g., "Are you hurting now?")

Understanding that elderly patients are less sensitive to pain

Recognizing that persistent pain is likely to affect physical and psychosocial functioning

Using the 0-10 pain scale, as it works well for nearly all older adults

Allowing extra time for the patient to assimilate the questions

Page 38: Pain Managment

Answer• Observing for changes in normal functioning

Asking about pain using synonyms, such as discomfort, aching, and sorenessFraming questions in the present tense (e.g., "Are you hurting now?")Recognizing that persistent pain is likely to affect physical and psychosocial functioningAllowing extra time for the patient to assimilate the questions

Page 39: Pain Managment

Persistent pain is common in older adults, particularly among the frail elderly, in whom cognitive impairment is more common. Age-related

changes in pain perception are probably not clinically significant. Functional changes, both psychosocial and physical, are common sequelae to chronic pain and may be the first indicators of pain in

cognitively impaired patients. A substantial portion of older adults (with and without cognitive impairment) have difficulty using the 0-10 pain

scale. Many other scales have demonstrated their validity in this population (Evidence level II). Many cognitively impaired older adults deny pain, but may be able to report distress when synonyms such as "aching" and "soreness" are used. Focusing on assessment of current symptoms (e.g., asking "Are you hurting right now?") may also help

those with short-term memory deficits.

Validated pain rating systems for the cognitively impaired focus on facial expressions, posture, vocalizations, appetite, and interactivity (Evidence

level II). Clinicians should choose one tool and use it consistently to ensure uniformity among health care providers (C level recommendation).

Page 40: Pain Managment

A 40-year-old female with three children has chronic low back pain and frequent tension headaches. In addition, she was recently treated for shoulder pain. Her neighbor has

suggested that she look into acupuncture and she asks you if acupuncture is safe and effective.

Which of the following would be accurate advice? (Mark all that are true.)

In a randomized study of chronic headache, those treated with acupuncture in addition to usual therapy had fewer headaches

than controlsThe addition of acupuncture to diclofenac (Voltaren) in patients with shoulder pain improves function more than

diclofenac aloneStudies that looked at more than 60,000 acupuncture

treatments showed no serious adverse events

Page 41: Pain Managment

Answer

• In a randomized study of chronic headache, those treated with acupuncture in addition to usual therapy had fewer headaches than controlsStudies that looked at more than 60,000 acupuncture treatments showed no serious adverse events

Page 42: Pain Managment

Acupuncture has been practiced for thousands of years and has been used for hundreds of different ailments. Studies of the method use sham treatments or minimal treatments as controls. Studies often show

conflicting results or small clinical effects. Acupuncture is quite safe, with no serious adverse effects reported in two studies including more than 60,000 treatments. Infection is minimized by using disposable needles

and aseptic technique. Serious bleeding is very rare.

In a meta-analysis of chronic back pain studies, acupuncture proved to be more effective than sham acupuncture or no treatment. For short-term pain relief in these patients it does not appear to be superior to other active therapies. It was not particularly effective in acute back pain

(Evidence level I). 

As an adjunct to usual therapies, acupuncture has proven effective in randomized studies of chronic headache and osteoarthritis of the knee (Evidence level I). It is used as an adjunct in cancer pain management.

One randomized, controlled trial of auricular acupuncture showed a positive effect in decreasing cancer pain when used with routine

analgesics.

Page 43: Pain Managment

A new patient comes to your office for evaluation of pain. The patient history should include which of the following?

(Mark all that are true.)Identification of possible pain generatorsA worker's compensation and litigation

historyA history of the onset and progression of

the painA complete medication history

A substance abuse history

Page 44: Pain Managment

Answer

• Identification of possible pain generatorsA worker's compensation and litigation historyA history of the onset and progression of the painA complete medication historyA substance abuse history

Page 45: Pain Managment

In the evaluation of pain, the history may be more valuable than the physical examination. An important goal of the encounter is to identify

the pain generator when possible, and the history may be the most illuminating part of the evaluation in this regard (C level

recommendation). Often the specific pain generator cannot be identified. History taking requires very active listening, with interplay between what the patient is saying and the physician's interpretation and clarification.

Obtaining a history of the onset and progression of the pain is of great importance. It can tell the physician whether this is an acute process and if

immediate action is needed (C level recommendation). It also provides clues as to the amount of additional history that will be needed to sort out previous treatment successes and failures. A history of legal action related

to pain, for example, is associated with a worse prognosis.

The medication history is a very important part of the initial evaluation. Rather than just a list of medications the patient is taking, it should

include a discussion of efficacy, tolerability, and economics (C level recommendation). It might also provide some idea of the patient's

attitudes toward medicines and expectations for efficacy. A history of substance abuse must be elicited because it has important implications in

the treatment plan and the need for safeguards.

Page 46: Pain Managment

A 30-year-old brick mason presents to your office with mid-back pain. On examination you note that his rhomboid

muscles are in spasm, and he jumps when you touch three discrete points in the muscles.

He is concerned that he may be developing fibromyalgia like his mother. True statements regarding the differentiation

between myofascial pain syndrome and fibromyalgia include which of the following? (Mark all that are true.)

The tender points of fibromyalgia are different from the trigger points seen with myofascial pain syndrome

Muscle spasm is most often associated with fibromyalgiaA jump/twitch response is most often associated with

myofascial pain syndromeThe tender points in fibromyalgia patients tend to be

distributed asymmetricallyMyofascial pain tends to be regional

Page 47: Pain Managment

Answer

• The tender points of fibromyalgia are different from the trigger points seen with myofascial pain syndromeA jump/twitch response is most often associated with myofascial pain syndromeMyofascial pain tends to be regional

Page 48: Pain Managment

The trigger points seen with myofascial pain syndrome are different from the tender points seen with fibromyalgia. Trigger points are discrete, focal, hyperirritable spots located in a taut band of skeletal muscle. Compression

of these points is painful and can produce referred pain, referred tenderness, motor dysfunction, and autonomic phenomena. Trigger points may be single or multiple, and are usually asymmetric. Pressing them may elicit a twitch in the muscle or a jump response from the patient. Trigger

points are associated with regional pain syndromes. Patients with fibromyalgia exhibit multiple tender points symmetrically distributed

along the axial skeleton, and have constitutional symptoms such as fatigue, sleep disturbance, and depressed mood.

No single modality stands out as the best for long-term treatment of trigger points and myofascial pain. However, trigger point injections are widely

accepted and recommended for providing short-term relief (C level recommendation).

Dry-needle techniques usually result in more soreness the next day than injection of local anesthetic. The addition of corticosteroids and other

medications to local anesthetics is unnecessary for efficacy and may cause muscle damage. The technique for trigger point injection is well described

in the reference article.

Page 49: Pain Managment

Common adverse effects of NSAIDs include which of the following? (Mark all that are

true.)Renal toxicity

Gastrointestinal bleedingPeripheral edema

Increased systolic blood pressure

Page 50: Pain Managment

Answer

• Renal toxicityGastrointestinal bleedingPeripheral edemaIncreased systolic blood pressure

Page 51: Pain Managment

NSAID use is associated with renal toxicity, gastrointestinal bleeding and ulcers, peripheral edema, and increased systolic blood pressure (median 5 mm

Hg) (Evidence level I).

Page 52: Pain Managment

A 30-year-old female at 38 weeks gestation comes to the hospital with irregular contractions. Her membranes ruptured spontaneously while she was at home. She has a history of sciatica, and underwent back surgery 2 years ago. Since then she has been taking opioids for pain control. She is

currently on sustained-release morphine, 15 mg three times daily, and acetaminophen/oxycodone (Percocet), 5 mg/325 mg every 4-6 hours as

needed for breakthrough pain.

The patient complains of low back pain with contractions but refuses epidural anesthesia. The resident on call orders nalbuphine (Nubain), 10 mg intravenously every 3 hours as needed for pain. After the first dose, the pain worsens and the resident approves a repeat dose. The patient

develops severe low back pain, nausea, vomiting, and tremors, and starts feeling very anxious.

What is the most likely cause of the patient's worsening symptoms? Underdosing of nalbuphine, as the patient is tolerant to opioids 

Increased intensity of labor contractions 

Overdosage due to giving a repeat dose of nalbuphine too soon 

Decreased opioid effect resulting from fetal absorption 

Withdrawal symptoms caused by nalbuphine antagonizing μ receptors

Page 53: Pain Managment

Answer

• Withdrawal symptoms caused by nalbuphine antagonizing μ receptors

Page 54: Pain Managment

Nalbuphine is an agonist/antagonist opioid medication. It has an agonist effect on κ receptors.

This is the reason it seems to work better for women, as they respond better to κ receptor-agonists. Men

respond better to μ receptor-agonists for pain control. Nalbuphine has an antagonist effect on μ

receptors, which is why it should not be used to treat pain when patients are on chronic opioid therapy

such as morphine (C level recommendation), which is a μ receptor-agonist. Nalbuphine's action is similar

to that of naloxone, and it will cause opioid withdrawal symptoms such as nausea, vomiting,

diarrhea, goose bumps, excessive yawning, tremors, runny nose, high blood pressure, and anxiety.

Page 55: Pain Managment

Cultural aspects of pain include which of the following? (Mark all that are true.)

The dominant culture of pain in the United States honors the stoical person

The meaning and expression of pain is influenced by the patient's culture

Persons from cultures different from that of their treating physician often receive

inadequate pain managementPain behaviors can be predicted reliably by

understanding a patient's cultureTo minimize bias, physicians must be aware

of their own pain experiences and culture

Page 56: Pain Managment

Answer

• The dominant culture of pain in the United States honors the stoical personThe meaning and expression of pain is influenced by the patient's culturePersons from cultures different from that of their treating physician often receive inadequate pain managementTo minimize bias, physicians must be aware of their own pain experiences and culture

Page 57: Pain Managment

Culture is the framework that directs human behavior in a given situation. The meaning and expression of pain are influenced by people's cultural background. Pain is not just a

physiologic response to tissue damage, but also includes emotional and behavioral responses based on individuals' past experiences and perceptions of pain. However, not everyone in every culture conforms to a set of expected behaviors or beliefs, so trying to categorize a person into a particular cultural stereotype will lead to inaccuracies. On the other hand,

knowledge of a patient's culture may help to better understand their behavior. 

Studies have shown that patients from ethnic minorities and cultures different from the health care professionals treating them receive inadequate pain management (Evidence level II). Each of us has the impression that people from distinct cultures are more or less likely to

express their pain experience in a manner that is somewhere between quietly enduring (stoical) or very expressive. While the physician should attempt to treat the expressive

patient and the stoical patient alike, physicians from a stoical culture are likely to be more attentive to the patient who is stoical. The culture of pain in mainstream American culture

tends to teach the hurting person to be stoical and the attending person to honor that stoicism.

For the physician, even more important than understanding the culture of others is understanding how his or her own upbringing affects attitudes about pain (C level

recommendation). It is important to overcome the belief that one's own reaction to pain is "normal" and that other reactions are "abnormal." Even subtle cultural and individual

differences between patient and physician, particularly in nonverbal, spoken, and written language, can affect care.

Page 58: Pain Managment

List three chronic conditions that may

be effectively treated by spinal cord stimulation.

Page 59: Pain Managment

Spinal cord stimulation, also known as dorsal column stimulation, was introduced in 1967. It has been applied successfully to a number of pain disorders including angina, tumors, brachial plexus injuries, spinal cord injuries, phantom limb pain,

complex regional pain syndrome/reflex sympathetic dystrophy, ischemic limb pain, multiple sclerosis, peripheral vascular disease, arachnoiditis, and failed back surgery syndrome. Success rates are variable. While spinal cord stimulation has

been utilized for a number of pain conditions, there are a limited number of randomized trials regarding its use (Evidence level I, Cochrane review). More

trials are necessary to confirm that spinal cord stimulation is an effective treatment for certain types of chronic pain.

Success Rates for Spinal Cord StimulationDiagnosis% SuccessFailed back surgery syndrome/low back & leg pain62

Ischemic limb pain77Complex regional pain syndrome I and II84

Peripheral neuropathy67Spinal cord injury57

Postherpetic neuralgia82Stump (phantom limb) pain62

Adapted from Cameron T: Safety and efficacy of spinal cord stimulation for the treatment of chronic pain: A 20-year literature review. J Neurosurg 2004;100(3

suppl Spine):264.Spinal cord stimulation is a late resort for chronic intractable pain conditions. The

risks and costs of spinal cord stimulation may outweigh the benefits for many patients.

Page 60: Pain Managment

A 58-year-old female with metastatic breast cancer has bone involvement. She is undergoing active

treatment and is still working despite her pain. She also has nausea from the chemotherapy.

True statements regarding this situation include which of the following? (Mark all that are true.)

As many as 80% of cancer patients with advanced or terminal cancer disease have bone metastases

Bone pain is usually sharp, and worsens with restingProstaglandin is thought to be involved in cancer-

related bone painIrradiation is not effective in the relief of bone pain

from metastasesRelief of bone pain by irradiation, if achieved, is

short-term in effect

Page 61: Pain Managment

Answer

• As many as 80% of cancer patients with advanced or terminal cancer disease have bone metastasesProstaglandin is thought to be involved in cancer-related bone pain

Page 62: Pain Managment

Metastatic bone pain is troublesome to patients trying to live and work with their disease. As many

as 84% of advanced or terminal cancer patients have bone metastases (Evidence level III). The pain is

aching in quality and worse with moving or bearing weight. Prostaglandins are thought to be involved in the pain, accounting for the surprisingly good results with irradiation. About 80% of patients treated with irradiation will have complete or substantial relief of their pain (Evidence level I, Cochrane review). Two-thirds of those will remain pain free in the irradiated

area for the rest of their lives. The role of prostaglandins is also the reason for the often surprising effectiveness of NSAIDs for relief.

Page 63: Pain Managment

A 52-year-old female with type 2 diabetes sees you for a routine follow-up. Her diabetes is controlled with diet, exercise, and oral

antihyperglycemics. While her diabetes has not always been under good control, her last hemoglobin A1c was 6.3%. Her cholesterol is under good

control and a recent stress test was negative.

At today’s visit, she describes a painful numbness in her toes bilaterally. She has been able to continue working but the pain is beginning to

interfere with her sleep. After performing a physical examination, you decide to treat her symptoms.

True statements regarding this situation include which of the following? (Mark all that are true.)

A low dose of a tricyclic antidepressant is the preferred initial therapySSRIs would be an appropriate first-line therapy if depression were also

presentThe efficacy of gabapentin is similar to that of amitriptyline

Opioids, alone or in combination with antidepressants, would be effective

Page 64: Pain Managment

Answer

• A low dose of a tricyclic antidepressant is the preferred initial therapyThe efficacy of gabapentin is similar to that of amitriptylineOpioids, alone or in combination with antidepressants, would be effective

Page 65: Pain Managment

Meta-analyses consistently show that tricyclic antidepressants (TCAs) are effective for neuropathic pain (Evidence level I, Cochrane review). They

can be of particular benefit where insomnia, anxiety, or depression is present. SSRIs are not considered first-line therapy for diabetic

neuropathy because the evidence of their effectiveness is limited (Evidence level I). Duloxetine and venlafaxine have demonstrated

efficacy in treating neuropathic pain (Evidence level II). An estimated 2.6 patients must be treated with TCAs and 6.7 patients with SSRIs to have

one patient with more than 50% pain relief. Gabapentin has a demonstrated efficacy in treating neuropathic pain (Evidence level I, Cochrane review). It is an alternative to TCAs where side effects or

contraindications prevent their use. A small randomized, controlled trial showed that gabapentin had an efficacy and tolerability similar to that of amitriptyline (Evidence level II). The efficacy of opioids in the treatment of neuropathic pain has been consistently demonstrated in randomized,

controlled trials, but they typically require greater caution than other options (Evidence level I).

Page 66: Pain Managment

A 70-year-old male has significant osteoarthritis in his knees. After failing conservative measures, he is evaluated for bilateral knee

replacement and expects to undergo surgery in the next few weeks. For pain management, you have prescribed acetaminophen, 1000 mg orally four times daily, but the patient is still having significant pain. You wish to improve his pain control with the use of an acetaminophen/opioid or

NSAID/opioid combination.

True statements regarding these medications include which of the following? (Mark all that are true.)

Combination medications may improve pain control while limiting the side effects associated with a higher dose of a single agent

Propoxyphene/acetaminophen (Darvocet N-100) provides pain control similar to that of acetaminophen alone

Codeine may be ineffective in up to 10% of African-American patients, due to a cytochrome P450 enzyme deficiency

When the maximum dosage of the acetaminophen or NSAID component is reached without sufficient pain relief, adding a pure opioid to the

nonopioid is recommendedUnsupervised use of over-the-counter medications along with combination medication increases the risk of adverse events

Page 67: Pain Managment

Answer

• side effects associated with a higher dose of a single agentPropoxyphene/acetaminophen (Darvocet N-100) provides pain control similar to that of acetaminophen aloneUnsupervised use of over-the-counter medications along with combination medication increases the risk of adverse events

Page 68: Pain Managment

The agents in combination medications operate through different mechanisms. Their use in combination can reduce the side effects of a higher dosage of a single agent. A meta-analysis of 26 trials involving 2,231 patients compared the

combination of acetaminophen and propoxyphene (Darvocet) to acetaminophen alone and found that the combination

provided little benefit over acetaminophen (Evidence level I). To become active, codeine is metabolized to morphine; it

may not be metabolized in up to 10% of Caucasians, due to a cytochrome P450 deficiency. Combination medications are

limited by their NSAID or acetaminophen content. When the maximum dosage is reached, switching to non-combination

medications is recommended (C level recommendation). Without express warnings, patients may use over-the-counter

medications that contain acetaminophen or NSAIDs. Unsupervised use of these medications increases the risk of

adverse events.

Page 69: Pain Managment

Many acute care and office procedures require anxiety control and/or pain control. True statements regarding sedation and pain control for procedures include which of the following? (Mark all that are

true.)Reported allergy to lidocaine (Xylocaine) is usually

due to the preservative methylparabenTo decrease injection pain, lidocaine should be buffered 10:1 with 8.4% sodium bicarbonate

The pain of injection can be decreased by using the smallest possible needle and injecting slowlyDiphenhydramine 1% (Benadryl) provides

anesthesia comparable to that produced by lidocaine

Page 70: Pain Managment

Answer• Reported allergy to lidocaine (Xylocaine) is

usually due to the preservative methylparabenTo decrease injection pain, lidocaine should be buffered 10:1 with 8.4% sodium bicarbonateThe pain of injection can be decreased by using the smallest possible needle and injecting slowlyDiphenhydramine 1% (Benadryl) provides anesthesia comparable to that produced by lidocaine

Page 71: Pain Managment

Guidelines for procedural sedation emphasize that the qualifications of the physician handling the procedure is the most important criterion when deciding whether to use procedural sedation. The physician must have an understanding of the medications

administered, must be able to monitor the patient's response to the medications, and must have the skills necessary to manage all potential complications (C level recommendation).

True allergy to the amine anesthetic lidocaine is rare, and a reaction is most likely due to the preservative methylparaben (Evidence level III). This can be circumvented by using preservative-free lidocaine; there is evidence that injection of 1% diphenhydramine

(Benadryl) solution gives anesthesia comparable to lidocaine injection, although it takes a bit longer to work. 

Topical anesthetics can be used to reduce the pain on initial local anesthetic injection, or topical anesthetics can be used in place of injection. Pain can also be reduced by using the

smallest needle that will work (30 gauge if possible), injecting very slowly, and lightly pinching the skin around the wound before and during the injection.

Regional nerve blocks are performed by using landmarks to guide the injection of local anesthetic in the potential spaces around the nerve supplying the area to be numbed. To

avoid systemic toxicity, one must avoid injecting into the arteries and veins in these spaces. Depending on the size of the nerve and the anesthetic used, it can take 5-20 minutes for a

block to become effective. Epinephrine can be used for local infiltration and field blocks, but never for nerve blocks. Using a longer-acting anesthetic such as bupivicaine in the nerve block will make it last longer, but it will take a few minutes longer to work. The blocked

region should be tested before beginning to incise, suture, or cauterize (C level recommendation).

Page 72: Pain Managment

A 37-year-old male presents to your office with a burn. While helping his wife with dinner yesterday, he tripped on a dog bone and put his left hand on a hot stovetop burner. When you examine the hand, you note that the wound covers the

entire palm, is red and blistered, and blanches under pressure. He took acetaminophen 2 hours ago, but still rates his pain as

8 on a scale of 10. He seems very uncomfortable.

After cleaning and dressing his wound, which one of the following would be most appropriate?

 No pain medication, because the affected nerve was destroyed 

Cold packs applied to the burn over the next 24-36 hours 

Aloe as needed and acetaminophen 4 times daily 

A neural blockade to stop the pain 

An opioid such as oxycodone/acetaminophen (Percocet)

Page 73: Pain Managment

Answer

• An opioid such as oxycodone/acetaminophen (Percocet)

Page 74: Pain Managment

Critique for this question:This patient has a partial thickness burn causing acute somatic pain that is unresponsive to

acetaminophen. An opioid such as oxycodone/acetaminophen will provide more potent analgesia and is suitable for this

pain profile (C level recommendation). In order to best treat a burn victim, it is necessary to distinguish the different levels of burns. This patient has a superficial partial thickness burn. These burns typically are red and blistered, although blisters may not appear for 12 hours after the injury. The skin will be

pink and moist under the blister, and the wound surface blanches with pressure.

Superficial burns can be treated with aloe and acetaminophen for pain. Neural blockade is generally used for neuropathic pain management. Although cold packs are initially good to keep swelling down, they are not suitable for extended pain

management of larger partial thickness burns.

Page 75: Pain Managment

Which one of the following is true regarding the use of glucosamine and chondroitin sulfate in the management of chronic osteoarthritis pain?

 Treatment with chondroitin sulfate is associated with a significant decrease in the incidence of joint

swelling, effusion, or both 

Glucosamine and chondroitin sulfate in combination works as fast as celecoxib (Celebrex) 

Glucosamine increases the risk of ischemic cardiovascular events for patients with diabetes

mellitus The use of glucosamine and chondroitin sulfate is

more beneficial for mild osteoarthritis pain than for moderate to severe pain

Page 76: Pain Managment

Answer

• Treatment with chondroitin sulfate is associated with a significant decrease in the incidence of joint swelling, effusion, or both

Page 77: Pain Managment

The dietary supplements glucosamine and chondroitin sulfate have been advocated, especially in the lay media, as safe and effective options for the management of symptoms of osteoarthritis. Glucosamine and chondroitin sulfate are the most

widely used dietary supplements for osteoarthritis, with estimated sales in 2004 approaching $730 million.

Several studies have evaluated the efficacy of glucosamine and chondroitin sulfate. Some of these studies have shown that these supplements are effective, but have been criticized as having flaws such as failure to adhere to the intention-to-treat principle, enrollment of small numbers of patients, potential bias related to sponsorship of the study by the manufacturers

of the dietary supplements, and inadequate masking of the study agent.

The Glucosamine/Chondroitin Arthritis Intervention Trial (GAIT) was a randomized, double-blind, controlled, multicenter trial sponsored by the National Institutes of Health. It was designed to rigorously evaluate the efficacy and safety of

glucosamine and chondroitin sulfate, separately and in combination, in the treatment of pain due to osteoarthritis of the knee. Both placebo and celecoxib were used as control agents.

The GAIT trial showed that glucosamine and chondroitin sulfate, alone or in combination, did not reduce pain effectively in the overall group of patients with osteoarthritis of the knee. Exploratory analyses suggested that the combination of glucosamine and chondroitin sulfate may be effective in the subgroup of patients with moderate to severe knee pain

(Evidence level I). 

Treatment with chondroitin sulfate was associated with a significant decrease in the incidence of joint swelling, effusion, or both. There was no increased risk of ischemic cardiovascular events among patients who also received celecoxib, or

among patients with diabetes who received glucosamine.

In the United States, glucosamine and chondroitin sulfate are regulated as dietary supplements and are not held to the more stringent standards for pharmaceuticals. Substantial variation exists between the content listed on the labels of these

products and the actual product. Because the GAIT trial was conducted under pharmaceutical rather than dietary supplement regulations, agents identical to the ones used in the study may not be commercially available (Evidence level

III).

In making therapeutic decisions, physicians and patients alike should be aware of data suggesting that celecoxib has a much faster time to response than glucosamine, chondroitin sulfate, or the two in combination. Continuing research is

needed to establish the potential efficacy and increase our understanding of the biology, pharmacology, and pharmacokinetics of these agents.

Page 78: Pain Managment

A 68-year-old female completed surgery, radiation, and chemotherapy for breast cancer 6 years ago. She has recently

developed recurrent breast cancer metastatic to bone. She does not want further radiation or chemotherapy, and her

oncologist thinks she has less than 6 months to live. She asks you to continue to be her primary physician as she enters

hospice care. 

True statements regarding the treatment of pain in this situation include which of the following? (Mark all that are

true.)The management of cancer pain requires specialty

consultationNSAIDs are contraindicated in cancer patients

Acute or escalating pain requires prompt medical attentionFor constant pain with exacerbation, the analgesic regimen

should include a routine baseline dose and breakthrough dosing

Addiction is rarely an issue in patients with terminal illness

Page 79: Pain Managment

Answer

• Acute or escalating pain requires prompt medical attention

• For constant pain with exacerbation, the analgesic regimen should include a routine baseline dose and breakthrough dosing

• Addiction is rarely an issue in patients with terminal illness

Page 80: Pain Managment

Cancer pain varies greatly between individuals and during different stages of the illness. The physician must therefore assess the intensity and

quality and type of pain and choose appropriate interventions. NSAIDs are quite useful in cancer pain syndromes, particularly bone pain.

Escalating pain that is not promptly addressed will require more drastic and more intense therapy than pain that is treated promptly (C level

recommendation). Many patients at the end of life have constant pain with exacerbations, requiring both a routine baseline dose and patient-

controlled dosing for breakthrough pain. A long-acting formulation is typically used for baseline dosing and a shorter, quicker-acting

formulation for acute exacerbations. Breakthrough doses are usually 10%-30% of the patient's usual daily dose (C level recommendation).

Addiction behaviors at the end of life are usually seen in active substance abusers whose addiction predates their terminal illness. Drugs also may be diverted by family members with active addiction. Terminal patients

who do not have active substance abuse problems will experience tolerance, but almost never display addictive behaviors.

Page 81: Pain Managment

True statements regarding physical dependence on opioids include which of the

following? (Mark all that are true.)Physical dependence develops in most

patients taking opioids on a regular basis for more than a few weeks

Physical dependence is a marker of addictionWithdrawal symptoms develop after abrupt

cessation of the opioidPhysical dependence explains why patients

take higher doses than prescribedPhysical dependence explains the symptoms

produced by administration of an opioid antagonist, such as naloxone (Narcan)

Page 82: Pain Managment

Answer

• Physical dependence develops in most patients taking opioids on a regular basis for more than a few weeksWithdrawal symptoms develop after abrupt cessation of the opioidPhysical dependence explains the symptoms produced by administration of an opioid antagonist, such as naloxone (Narcan)

Page 83: Pain Managment

The American Pain Society, the American Academy for Pain Management, and the American Society of Addiction Medicine have

jointly created definitions for both physical dependence and addiction. Physical dependence is defined as "a state of adaptation that is manifested

by a drug class-specific withdrawal syndrome that can be produced by abrupt cessation, rapid dose reduction, decreasing blood level of the drug, and/or administration of an antagonist." It is an expected consequence of chronic opioid use and is distinct from addiction, which is defined as "a primary, chronic, neurobiologic disease, with genetic, psychosocial, and environmental factors influencing its development and manifestations. It is characterized by behaviors that include one or more of the following: impaired control over drug use, compulsive use, continued use despite

harm, and craving" (C level recommendation).

Withdrawal symptoms may be provoked by sudden cessation of the opioid, rapid dose reduction, malabsorption or metabolic changes leading

to reduced levels, and/or administration of an antagonist. Physical dependence does not explain why a patient would take a higher dose than

what was prescribed (Evidence level III).

Page 84: Pain Managment

True statements regarding the use of the fentanyl transdermal patch (Duragesic)

include which of the following? (Mark all that are true.)

According to the FDA, it should not be used for postoperative pain

It should be prescribed at the lowest dose needed when used on an as-needed basis

Patients may have a sudden rise in their body level of fentanyl if they become hypothermicBecause fentanyl is absorbed transdermally, other medications do not affect its plasma

levels

Page 85: Pain Managment

Answer

• According to the FDA, it should not be used for postoperative pain

Page 86: Pain Managment

The FDA is investigating reports of death and other serious adverse events related to narcotic overdose in patients using the fentanyl transdermal patch for pain

control. In June 2005 the Duragesic product label was updated to add new safety information in several areas of labeling, and a "Dear Healthcare Professional"

letter about these changes was issued by the manufacturer.

The directions for use of the fentanyl transdermal patch must be followed exactly to prevent death or other severe side effects from overdosing. These directions are

provided in the product label and patient package insert.

The fentanyl transdermal patch is a long-acting medication and should not be used for intermittent pain, short-term pain, or postoperative pain (C level

recommendation). Pain after an operation improves with time and requires a shorter-acting pain medication for PRN use and for easier titration downward. The

patch delivers a steady level of medication around the clock and cannot be used for breakthrough pain.

Patients on the fentanyl patch may have a sudden increase in their fentanyl plasma concentration if they have an increase in their body temperature or are exposed to

heat or use other medicines that increase the elimination half-life of fentanyl. Ketoconazole is a cytochrome P450 3A4 inhibitor, which is one type of drug that

can have this effect. Compared to other opioids, fentanyl patches are relatively expensive and are less flexible in terms of dosage titration.

Page 87: Pain Managment

A 5-year-old male has sustained a 3-cm laceration on his forehead that will require suturing. He is distraught and his

mother is having difficulty getting him to hold still to be examined. He is afraid of needles and the mother asks if you can “knock him out” to do the procedure. The boy is allergic

to penicillin.

True statements regarding the relief of pain and anxiety in this patient include which of the following? (Mark all that are

true.)A topical mixture of lidocaine, epinephrine, and tetracaine

(LET) can anesthetize the wound without injectionEutectic mixture of local anesthetics (EMLA) cannot be used

because of the patient's penicillin allergyVapo-coolant sprays will effectively control the pain of

intravenous injection for this patientCombining sedation and analgesia for this patient will require

a dedicated, trained observer and a physician skilled in pediatric airway management

Page 88: Pain Managment

Answer

• A topical mixture of lidocaine, epinephrine, and tetracaine (LET) can anesthetize the wound without injectionCombining sedation and analgesia for this patient will require a dedicated, trained observer and a physician skilled in pediatric airway management

Page 89: Pain Managment

American Academy of Pediatrics guidelines recommend using topical anesthetics prior to minor procedures (C level recommendation). Eutectic

mixture of local anesthetics (EMLA) and liposomal 4% lidocaine (LMX4) are effective in numbing intact skin to the pain of venipuncture

(including starting an intravenous line), lumbar puncture, joint aspiration, and abscess drainage (Evidence level I). EMLA's effect peaks at 1 hour, while LMX4 takes 30 minutes (Evidence level I). EMLA cannot be used

in patients who have recently taken sulfonamides, but is safe for those with penicillin allergy. For lacerations, lidocaine/epinephrine/tetracaine

(LET) can be placed in the wound. In about 10 minutes, LET can anesthetize a skin wound up to 5 cm long with good effect for 20-30

minutes (Evidence level I). 

While vapocoolant sprays decrease the pain of injections, they do not last long enough to affect the pain of intravenous line insertion (Evidence

level I). For prolonged procedures a combination of analgesia and sedation may be necessary. Each office or institution must have a policy

regarding this practice. A trained observer must focus on the child's breathing and circulation (C level recommendation). In addition, a

physician skilled in pediatric airway management must be immediately available.

Page 90: Pain Managment

True statements regarding the use of methadone for chronic non-cancer pain include which of the

following? (Mark all that are true.)Because of methadone's long half-life (>20 hours), it

should be avoided in elderly patientsMethadone's analgesic effect builds over 5-7 days, producing a duration of analgesia nearly equivalent

to its half-lifeMethadone is metabolized principally by the liver and should not be used in patients with hepatitis C

Methadone, unlike morphine, does not require adjustment for renal disease

Methadone can be used safely in patients allergic to morphine

Page 91: Pain Managment

Answer

• Methadone, unlike morphine, does not require adjustment for renal diseaseMethadone can be used safely in patients allergic to morphine

Page 92: Pain Managment

Methadone is a synthetic opioid with a half-life of about 22 hours. The duration of analgesia is shorter than the half-life,

but typically builds to 8-12 hours with repeated dosing. Methadone accumulates in the tissues, and serum levels build

gradually over 5-7 days. Dosing adjustments should not be made more frequently than every 5-7 days in the outpatient

setting (C level recommendation).

While methadone is metabolized by the liver, it is usually unnecessary to adjust methadone doses for hepatic disease. A minor portion of methadone is cleared by the kidneys and it is

usually unnecessary to adjust dosing for renal failure. Because methadone is synthetic, it can be used in patients with a true allergy to morphine. Methadone should be used with caution in the elderly patient, and a lower initial dose

may be appropriate (C level recommendation).

Page 93: Pain Managment

A 67-year-old male has recurrent prostate cancer with metastasis to his lumbar spine. His life expectancy is estimated to be 3 months. He has already received his maximum dosage of radiation and has elected to

receive only palliative therapies. He is currently using oral morphine and several adjuvants for pain control. Over the past several weeks, the

amount of morphine required to adequately control his pain is causing excessive sedation. You are considering a trial of neuraxial (epidural/intrathecal) administration of pain medications.

True statements regarding this situation include which of the following? (Mark all that are true.)

Neuraxial administration will maintain analgesia while decreasing sedation

Side effects such as pruritus, nausea, and urinary retention are rare with intrathecal administration

When neuraxial medications are used, rescue (breakthrough) doses should be administered by the same route

Due to the risk of infection, neuraxial techniques are limited to inpatient use

Page 94: Pain Managment

Answer

• Neuraxial administration will maintain analgesia while decreasing sedation

Page 95: Pain Managment

For uncontrolled pain or intolerable opioid side effects, neuraxial (epidural/intrathecal) administration of opioids

should be considered (level C recommendation). Opioids and local anesthetics can be delivered more efficiently to opioid receptors, providing analgesia while reducing systemic side

effects. Side effects may still include sedation, nausea, vomiting, pruritus, constipation, urinary retention, and

respiratory depression. Rescue (breakthrough) medications may be delivered by any effective route. After an appropriate

trial with a temporary delivery system, neuraxial administration may be continued using an implanted catheter and pump, or by using percutaneous catheterization and an

external pump. While the risk of infection exists, the literature suggests that home parenteral therapy with

appropriate support is effective for analgesia, with no notable risk of adverse effects.

Page 96: Pain Managment

Goals for treatment of chronic pain include which of the following?

(Mark all that are true.)A return to full-time employment

statusImproved physical function

Complete relief of painBetter quality of life

Improved function in family and social roles

Empowerment of the patient

Page 97: Pain Managment

Answer

• Improved physical functionBetter quality of lifeImproved function in family and social rolesEmpowerment of the patient

Page 98: Pain Managment

Empowerment of the patient is the overarching goal of therapy for chronic pain. The patient is the primary expert on the pain and its response to treatment, as well

as what constitutes realistic goals for therapy. Patient self-report is the most reliable indication of the presence and intensity of pain (C level recommendation), and similarly, the patient provides critical input about the level of pain reduction needed for improved quality of life. Complete relief of chronic pain is seldom a realistic goal, although reduction in its severity is obviously an important goal. This should be made clear in the initial partnership with the patient to address

chronic pain.

Improving physical function is another major goal of treatment; explicit steps toward this end should be negotiated with the patient (C level recommendation).

However, improved physical function will not translate to a full return to work for many chronic pain patients. Requiring it as a mark of success (or to continue to

"deserve" opioids) would be a mistake.

Depression is a common response to chronic pain, and preexisting depression may be a risk factor for its development (C level recommendation). All patients with

chronic pain should be assessed for this, along with other psychological comorbidities. Family issues also commonly arise and should be addressed.

Improved function in family and other social roles is an important goal to include in the treatment plan for chronic pain (C level recommendation). Input from other

family members (particularly the spouse) may be quite helpful in assessing the response to treatment of chronic pain.

Page 99: Pain Managment

You are seeing a patient, new to your practice, for failed back surgery syndrome. He reports he has failed a number of medications and

interventions and requests a prescription for sustained-action oxycodone (OxyContin). He says that his previous physician had prescribed 20 mg

orally three times a day, and this was providing good relief. He willingly completes a release which you promptly fax to his previous provider. He

also consents to urine drug testing, part of your office policy for new patients. In addition to the oxycodone, he reports he is using diazepam

(Valium) as needed for back spasms, and an over-the-counter cold medication. He denies using any illicit substances.

The results of the immunoassay point–of–care testing tool used by your office are shown below:

SubstanceCutoff (ng/mL)ResultMarijuana50NegativeCocaine300PositiveOpioids300NegativePhencyclidine25NegativeAmphetamines1000PositiveBenzodiazepines

200NegativeTrue statements regarding these results include which of the following?

(Mark all that are true.)The negative test for opioids rules out oxycodone use

The negative test for benzodiazepines rules out diazepam useThe positive test for cocaine indicates cocaine use within the past 48

hoursThe positive test for amphetamines indicates amphetamine use within the

past 48 hours

Page 100: Pain Managment

Answer

• The positive test for cocaine indicates cocaine use within the past 48 hours

Page 101: Pain Managment

Urine drug testing is often used to assist in the diagnosis of drug abuse/addiction prior to starting opioid therapy and to assist in monitoring

compliance with chronic opioid therapy. Immunoassay testing relies on the principle of competitive binding and uses antibodies to detect the

presence of a particular drug. Their advantage is the ability to simultaneously and rapidly test for drugs in urine. When urine drug

testing results are used for legal or employment purposes, the immunoassay results should be confirmed by mass spectroscopy/gas

chromatography. Immunoassays may only detect certain classes of drugs and may cross-react with other substances, producing inaccurate results. Immunoassays reliably detect morphine and codeine but do not reliably detect synthetic and semi-synthetic opioids (e.g., oxycodone, fentanyl,

methadone).

The test for cocaine reacts with both cocaine and its primary metabolite, benzoylecgonine. These tests have low cross-reactivity and are very

specific in predicting cocaine use. The tests for amphetamine/methamphetamine are highly cross-reactive. They will

detect other sympathomimetic amines such as ephedrine and pseudoephedrine. The negative result for benzodiazepines may indicate that the patient has not recently used the medications, or perhaps the test

was not sufficiently sensitive to detect the medication at the concentration present.

Page 102: Pain Managment

A 20-year-old male skateboarder presents to the emergency department with severe pain from a broken ankle. You decide to give him parenteral

pain medication but his chart lists an allergy to morphine. When you question him about this, he says that the last time he had a broken bone he received morphine and it made him nauseated and he had severe itching

all over.

When determining whether to prescribe an opioid to this patient, which of the following should be kept in mind? (Mark all that are true.)

Most true allergic reactions to opioids involve codeine, morphine, or meperidine (Demerol)

Opioid side effects such as nausea, itching, and mild hypotension are often mistaken for true allergy symptoms

True opioid allergy is a class effect, so allergy to one drug eliminates the use of all other opioids

Angioedema, severe hypotension, and bronchospasm are signs of true opioid allergy

Tramadol (Ultram) is a safe substitute for patients with true opioid allergy

Page 103: Pain Managment

Answer

• Most true allergic reactions to opioids involve codeine, morphine, or meperidine (Demerol)Opioid side effects such as nausea, itching, and mild hypotension are often mistaken for true allergy symptomsAngioedema, severe hypotension, and bronchospasm are signs of true opioid allergy

Page 104: Pain Managment

Side effects such as nausea, constipation, mild hypotension, and itching are often mistaken for signs of opioid allergy. True allergy is very rare. Urticaria and itching are often seen with initial doses of opioids. This is from mast cell release of histamine, and is not IgE-mediated. It can be treated with antihistamines.

When patients have angioedema, severe hypotension, or bronchospasm with opioid use, it must be considered a true allergy. A drug from a different opioid class can be used, but there are reports of

individuals who are allergic to more than one class of opioids (C level recommendation). Tramadol is also contraindicated in the presence of true opioid allergy.

Opioid Chemical Classes

PhenanthrenesMorphineCodeine

Hydrocodone (Vicodin, Lortab)Oxydcodone (Percocet, Tylox, OxyContin)

Oxymorphone (Numorphan) Hydromorphone (Dilaudid)

Nalbuphine (Nubain)Butorphanol (Stadol)

Levorphanol (Levo-Dromaran)Pentazocine (Talwin)

PhenylpiperadinesMeperidine (Demerol)

Fentanyl (Duragesic, Actiq, Sumlimaze)Sufentanil (Sufenta)

Remifentanyl (Ultiva)

DiphenylheptanesMethadone (Dolophine)

Propoxyphene (Darvon, Darvocet)

Page 105: Pain Managment

A 40-year-old male has chronic pain after three back operations. In discussing his overall health you discover that he gave up walking a year ago because of pain, and he has gained 10 lb in the last year. He asks you

if relaxation therapy would help his pain.

You consider the use of exercise, relaxation, or behavioral treatments to manage the patient's chronic back pain. True statements regarding this

situation include which of the following? (Mark all that are true.)A Cochrane review found that interdisciplinary biopsychosocial rehabilitation for >100 hr decreased pain and improved function

Multiple meta-analyses have shown that cognitive therapy modestly reduces chronic back pain

Passive modalities for back pain, such as TENS, relaxation, and massage, are recommended in place of active exercise regimens for patients with

limited mobilityLearning to use the relaxation response has been shown to reduce clinic

visits by chronic back pain patients

Page 106: Pain Managment

Answer• A Cochrane review found that interdisciplinary

biopsychosocial rehabilitation for >100 hr decreased pain and improved functionMultiple meta-analyses have shown that cognitive therapy modestly reduces chronic back painLearning to use the relaxation response has been shown to reduce clinic visits by chronic back pain patients

Page 107: Pain Managment

Multiple studies and several guidelines agree that exercise and fitness are a key component in relieving chronic back pain. No particular type of

exercise demonstrates a clear advantage over any other. Guidelines recommend fostering self-management skills in patients and setting goals

for functional improvement (Evidence level C). Only very intense interdisciplinary biopsychosocial interventions made a difference when reviewed by the Cochrane group (Evidence level I, Cochrane review). A recent clinical inquiry confirmed the modest effect of cognitive therapies for improving some chronic pain states, and emphasized the efficacy of

tricyclic antidepressants in reducing chronic back pain (A level recommendation).

Meditative therapies such as Benson's relaxation response (repetition of a word, phrase, prayer, or other activity to counteract stress) have been

studied in a variety of contexts. The studies vary in strength of response, numbers, and quality of controls. It is difficult to make any strong

recommendations for chronic pain, but such therapies are relatively low cost and harmless (Evidence level III).

Page 108: Pain Managment

True statements regarding the management of chronic daily headaches include which of the

following? (Mark all that are true.)A small daily dose of prednisone (5-10 mg) helps decrease the frequency of headaches

Amytriptyline can reduce headache frequency by up to 50%

Opioids are effective (50% improvement) in more than 60% of patients

NSAIDs are associated with a lower risk of medication overuse headaches compared to

ergotamine

Page 109: Pain Managment

Answer

• Amytriptyline can reduce headache frequency by up to 50%NSAIDs are associated with a lower risk of medication overuse headaches compared to ergotamine

Page 110: Pain Managment

Chronic daily headache refers to the presence of a headache more than 15 days per month for longer than 3 months. Chronic daily headache is not a diagnosis but a category that contains many disorders representing primary and secondary headaches. Secondary causes must be ruled out before the diagnosis of a primary headache disorder is made. Approximately 3% -5% of the population worldwide and 70%-80% of patients presenting to headache clinics in the United States have daily or near-daily headaches. The

disability associated with this disorder is substantial and includes a diminished quality of life related to physical and mental health, as well as impaired physical, social, and occupational functioning.

The overuse of medications used for acute headache may lead to medication overuse headache, a syndrome of daily headaches caused by the very medications used to relieve the pain. The prevalence in the population of chronic daily headache associated with overuse of these medications was recently estimated to be 1.4% overall, with a higher estimated occurrence among women (2.6%), especially

those over the age of 50 years (5%) (Evidence level II).

Overuse of medications for acute headache is defined as any of the following:

    regular overuse of a headache medication for >3 months    use of ergotamine, triptans, opioids, and combination analgesics >10 days/month

    use of simple analgesics 15 or more days/month    use of any headache medications 15 or more days/month

NSAIDs and dihydroergotamine mesylate (unlike ergotamine tartrate) are generally associated with a low risk of medication overuse headache, and are often used to treat breakthrough headaches during the withdrawal period.

Randomized trials of the use of preventive medications in chronic daily headache are scarce. In a single trial involving amitriptyline, the frequency of headache was reduced by more than 50% in over half of the study participants (Evidence level I).

A recent double-blind, placebo-controlled study evaluated the effect of 100 mg of prednisone for 5 days on the duration of severe withdrawal headache in 20 patients with presumed medication overuse headache. There was a significant reduction in the number of hours of severe withdrawal headache in the active-treatment group, which confirmed earlier observations from uncontrolled studies

(Evidence level I).

The use of daily opioid therapy in patients with chronic daily headache is controversial. A recent prospective study with an initial cohort of 160 patients who were prescribed daily opioid therapy reported the outcomes among 70 patients with medically refractory

chronic daily headache who continued this therapy for at least 3 years. Only 41 of the original 160 patients (26%) had an improvement of 50% or more on a headache index that took into account the frequency and severity of headaches each week (Evidence level II). Half of the patients had "problem drug behavior" (defined as "lost" prescriptions, seeking medication from other sources, and most

commonly, dosage violations). Most of these patients (74%) either did not show marked improvement or were dropped from the program because of problem drug behavior. These data underscore the low efficacy of long-term opioid therapy and the high risk of

misuse in this patient population.

Page 111: Pain Managment

A patient at 32 weeks gestation asks about her options for the management of labor pain.

Appropriate advice would include which of the following? (Mark all that are true.)Epidural anesthesia is associated with increased rates of cesarean delivery

Epidural anesthesia is associated with increased instrumentation rates at delivery

Continuous labor support decreases maternal pain during labor

Warm water baths can decrease labor pain for short periods of time

Page 112: Pain Managment

Answer

• Epidural anesthesia is associated with increased instrumentation rates at deliveryContinuous labor support decreases maternal pain during laborWarm water baths can decrease labor pain for short periods of time

Page 113: Pain Managment

Two large meta-analyses have shown little effect of epidural anesthesia on cesarean delivery rates. Epidural anesthesia

provides better pain relief in labor than opioids, but is associated with increased instrumentation rates, maternal

fever, and a slightly longer duration of labor (Evidence level I). A randomized, controlled trial of intrathecal fentanyl

showed superior pain relief and shorter labor duration with no increase in cesarean delivery rates compared to systemic

hydromorphone (Evidence level I). Reviews of continuous labor support with doulas show decreased rates of operative

delivery and decreased requests for pain medication (Evidence level I). Warm water baths appear to decrease pain but have a short duration of action (Evidence level I). Sterile saline injections provide short-term relief of back pain but do not decrease requests for pain medication (Evidence level I). Despite years of use there remains little clear evidence on the

safety and efficacy of opioids in labor (Evidence level I).

Page 114: Pain Managment

Which one of the following is true regarding the use of agreements or

contracts for patients using opioids for chronic pain?

 Written agreements reduce the rate of addiction and abuse

 Contracts provide the physician with legal protection 

Contracts are often recommended by experts in chronic pain management 

Contracts have been shown to improve the patient-physician relationship

Page 115: Pain Managment

Answer

•  Contracts are often recommended by experts in chronic pain management 

Page 116: Pain Managment

Contracts or formal agreements are frequently recommended by experts in chronic pain management, discussed extensively in related literature, and used by many physicians. While certain goals of contracts could be supported by virtually all (clarifying plans for use, providing informed consent, and reducing the risk of addiction), consideration of their use

should include an understanding of their potential negative consequences. They may damage the patient-physician relationship, erode the patient's sense of trust and reliance on the physician's beneficence, or provide a

false sense of security for both the patient and the physician about the risk of addiction (C level recommendation).

Finally, no studies have demonstrated a reduction in the incidence of addiction or abuse when contracts are used (Evidence level III).

Physicians should be familiar with the legal requirement of their own states, but the Federation of State Medical Boards model policy on the use of controlled substances for pain states that the physician should consider a written agreement for patients at high risk for medication abuse, or for

those with a history of substance abuse (C level recommendation).

Page 117: Pain Managment

A 73-year-old male who is dying of metastatic lung cancer has been taking sustained-release morphine, 120 mg twice

daily, with occasional use of immediate-release morphine, 60 mg, for breakthrough pain. His pain has been well controlled by this regimen, but as he nears death he is no longer able to

swallow.

Appropriate strategies for continuing to manage his pain include which of the following? (Mark all that are true.)Continue the same dosing regimen of morphine by rectal

suppositorySwitch to sublingual morphine, 40 mg every 4 hours

Switch to an equianalgesic dose of methadone for easier administration

Convert to an equianalgesic dose of fentanyl (Duragesic) (transdermal and sublingual)

Begin a subcutaneous morphine infusion

Page 118: Pain Managment

Answer

• Continue the same dosing regimen of morphine by rectal suppositorySwitch to sublingual morphine, 40 mg every 4 hoursConvert to an equianalgesic dose of fentanyl (Duragesic) (transdermal and sublingual)Begin a subcutaneous morphine infusion

Page 119: Pain Managment

The preferred route of administration of analgesia for most patients is the oral route. However, when that route is no longer viable, other routes should be considered.

Limited data, along with expert consensus, suggest that oral opioids, both sustained-release and immediate-release forms, may be given rectally for equivalent analgesia. (These equivalencies are

generally considered to be true for morphine, hydromorphone and oxycodone. More studies have been conducted on morphine than the other opioids. Methadone has little data available regarding such

equivalencies.) (C level recommendation.)

This is a simple solution for many patients. However, family members may have both physical and psychological difficulties in performing rectal administration, and these should be considered.

The pharmacokinetics of sublingual or buccal administration are similar to those of oral administration for immediate-release preparations. However, sustained-release preparations cannot be administered by

this route.

An opioid infusion, either intravenous or subcutaneous, may also provide a good alternative for selected patients. In general, the equianalgesic dosage for parenteral administration will be approximately one-

third of the oral dose (Evidence level III). Further reductions may be indicated for the starting dose if it is thought that the oral medications were not being well absorbed.

A transdermal patch, available only for fentanyl at this time, may also be considered, but it is important to remember that this will not reach a steady state until approximately 18 hours after application. Patients

will require other means of analgesia in the interim and until the correct dosage is established (C level recommendation). Equianalgesic ranges for fentanyl patches are quite wide. Dying patients may also have alterations in peripheral circulation and subcutaneous reservoirs that make transdermal patches a

less reliable approach.

Page 120: Pain Managment

True statements regarding the management of fibromyalgia include which of the following? (Mark all that are true.)Tricyclic antidepressants or cyclobenzaprine (Flexeril) at

bedtime is an appropriate initial therapyNSAIDs may be used effectively either as monotherapy or in

combination with other medicationsAerobic exercise two to three times per week may improve

conditioning and fibromylagia symptomsSleep and antianxiety agents such as trazodone (Desyrel),

benzodiazepines, and nonbenzodiazepine sedatives are indicated if sleep disturbance is a prominent symptom

Multidisciplinary approaches that incorporate two or more strategies help decrease pain and improve function

Page 121: Pain Managment

Answer• Tricyclic antidepressants or cyclobenzaprine (Flexeril) at

bedtime is an appropriate initial therapyAerobic exercise two to three times per week may improve conditioning and fibromylagia symptomsSleep and antianxiety agents such as trazodone (Desyrel), benzodiazepines, and nonbenzodiazepine sedatives are indicated if sleep disturbance is a prominent symptomMultidisciplinary approaches that incorporate two or more strategies help decrease pain and improve function

Page 122: Pain Managment

The following recommendations regarding the management of fibromyalgia are supported by the strongest evidence (Strength of evidence A: There is evidence of type I, or consistent findings from multiple studies of types II, III, IV).

Evaluation of the patient with fibromyalgia syndrome (FMS) begins with a complete history and physical examination, focusing on illnesses that may mimic or complicate FMS, such as hypothyroidism or ankylosing spondylitis, or that can

occur concurrently with FMS, such as tendinitis, systemic lupus erythematosus, rheumatoid arthritis, or osteoarthritis. The clinician should perform a complete joint examination, manual muscle strength testing, and a neurologic examination. 

The clinical diagnosis of FMS depends on the presence of widespread pain, defined as pain in all four body quadrants and axial pain, for at least 3 consecutive months. The only physical examination criterion for the diagnosis of FMS is the

presence of excess tenderness to manual palpation of at least 11 of 18 muscle-tendon sites.

Multiple strategies, including both pharmacologic and nonpharmacologic therapies, should be used in the management of FMS. For initial treatment of FMS, a tricyclic antidepressant, in particular 10-30 mg amitriptyline, or cyclobenzaprine can be given at bedtime to promote sleep. An SSRI such as fluoxetine, alone or in combination with a tricyclic, can be used for

pain relief. NSAIDs should not be used as the primary pain medication. There is no evidence that NSAIDs are effective when used alone, although NSAIDs (including COX-2 selective agents) and acetaminophen may provide some analgesia

when used with other medications.

Other potentially useful medications include sleep and antianxiety medications such as trazodone, benzodiazepines, nonbenzodiazepine sedatives, or L-dopa and carbidopa, especially if sleep disturbances such as restless legs syndrome are

prominent.

Patients with FMS should be encouraged to perform moderately intense aerobic exercise (60%-75% of age-adjusted maximum heart rate) two to three times per week. In individuals who are deconditioned, this rate can be achieved with very

low levels of exercise.

Multidisciplinary approaches incorporating two or more strategies decrease pain and improve function in FMS, especially in people who have not responded to simpler approaches.

Page 123: Pain Managment

True statements regarding the use of tramadol (Ultram) include which of

the following? (Mark all that are true.)

It is a μ opioid receptor agonistIt may lower the seizure threshold

It is an effective agent for neuropathic pain

It can cause serotonin syndrome when used with SSRIs

Page 124: Pain Managment

Answer

• It is a μ opioid receptor agonistIt may lower the seizure thresholdIt is an effective agent for neuropathic painIt can cause serotonin syndrome when used with SSRIs

Page 125: Pain Managment

Tramadol is a weak μ agonist. It also causes both norepinephrine and serotonin reuptake inhibition and has the potential to lower the seizure threshold. In a surveillance study, the risk of seizure was increased two- to sixfold among users, adjusted for selected comorbidities and concomitant drug use (Evidence

level III). The risk was highest among those aged 25-54 years, those with more than four tramadol

prescriptions, and those with a history of alcohol abuse, stroke, or head injury. There is also a risk of

serotonin syndrome when tramadol is used with SSRIs. Tramadol has demonstrated benefits in

neuropathic pain (Evidence level I). Tramadol's side effects include nausea, constipation, and dizziness. Its side effect profile is similar to that of codeine.

Page 126: Pain Managment

A 62-year-old female with metastatic cancer of the colon is expected to die within weeks. She has had

pain in her right chest where a large pulmonary metastasis has been identified. This pain has been

well controlled with sustained-release morphine, 60 mg twice daily.

What is the most appropriate dose of immediate-release morphine for breakthrough pain every 1-2

hours on an as-needed basis? 10 mg 

30 mg 

60 mg 

120 mg

Page 127: Pain Managment

Answer

•30 mg 

Page 128: Pain Managment

Most experts recommend a starting dose for breakthrough pain of 10%-25% of the total daily dose, to be adjusted according to the

patient's response. An oral opioid dose may be repeated after 1 hour if the response is

inadequate. The patient should be instructed that recurrent need for more than one dose per

episode of pain is an indication for dosage adjustment, and the physician should be contacted. Similarly, a regular need for

breakthrough medication suggests that the sustained-release dosage should be

reassessed.

Page 129: Pain Managment

A 39-year-old male presents to your office with an acute onset of low back pain. The pain started 2 days ago when he twisted to get out of his

car at the end of a 4-hour drive. He rates the pain as 7 on a 10-point scale, and describes it as dull and aching. He gets mild relief by lying down and finds that sitting exacerbates it. The pain is mainly in the left paraspinal

area from L3-L5 with no radiation, and tenderness in that area is the only abnormal physical finding.

The patient asks your opinion about seeing the chiropractor one of his friends recommended. Which one of the following would you tell him

about the benefits of manipulation for this problem? Spinal manipulation is more likely to shorten the duration of his pain than

any other treatments you may prescribe Spinal manipulation is more likely to reduce the severity of his pain than

any other treatments you may prescribe 

Spinal manipulation combined with acupuncture has been found to be the most effective treatment for his type of pain

 Spinal manipulation does not offer any lasting advantages over the other treatments you usually prescribe

Page 130: Pain Managment

Answer

• Spinal manipulation does not offer any lasting advantages over the other treatments you usually prescribe

Page 131: Pain Managment

Acute low back pain is an extremely common problem with high costs of care. Spinal manipulation therapy is often recommended, despite contradictory

evidence regarding its effectiveness.

A 2004 Cochrane review concluded that spinal manipulation was more effective than sham

(placebo) therapy for reducing the severity and duration of pain, and for improving functional

ability. However, it has not been shown to be more effective than pain medication, physical therapy,

exercise, back school, or usual primary care (Evidence level I, Cochrane review).

Page 132: Pain Managment

A 72-year-old male is brought to your office by his daughter for a routine follow-up visit. He is blind and has type 2

diabetes. He is taken care of at home by the daughter. He states that he is doing “okay” and that his peripheral

neuropathy and activity level have remained the same since his last visit 3 months ago. His daughter seems frustrated and reports that his pain is worse, and that his level of activity has decreased. When asked to give an example, she says, "Trust

me, his pain is worse." Your examination reveals nothing remarkable, and the patient’s HbA1c level is 6.3%.

You decide tohave the patient and daughter start a patient pain and activity log

prescribe an anxiolytic for the daughter to relieve her stressincrease the patient's neuropathic pain medication

Page 133: Pain Managment

Answer

• have the patient and daughter start a patient pain and activity log

Page 134: Pain Managment

The issues of communication, social relationships, psychological well-being, caregiving needs, and spirituality combine to define a complex set

of roles, experiences, and perspectives in the care of loved ones. Frequently, caregivers report higher levels of pain and immobility than

the patient. On the other hand, patients may be hesitant to admit that they need more care (Evidence level II). They may understate their pain due to

an increased concern for caregiving needs and future dependency. 

Scheduling time with the daughter would help her better understand the signs of pain, and the factors that influence how patients report their pain (Evidence level III). A way to obtain a more accurate representation of

the patient's health would be to create a daily patient pain and activity log. Meeting with the daughter would also facilitate a review of the health

system to ensure that she has the community and psychological support services that she needs (Evidence level III). This can help prevent anxiety

and depression and reduce unnecessary prescribing of medications. 

While there are conflicting reports of pain and activity levels, the evaluation reveals good control of the patient's glucose levels. In this

case, increasing pain medication would not be indicated. Neither of these two decisions addresses the patient-caregiver relationship.

Page 135: Pain Managment

A 30-year-old male with AIDS-related neuropathy is experiencing incomplete pain control. He asks you if

dronabinol (Marinol) or another cannabinoid is likely to help his pain. Which one of the following

would be accurate advice? Oral cannabinoids are not associated with CNS

depression or psychotropic effects 

Cannabinoids such as dronabinol are no more effective than codeine 

Dronabinol has been shown to be effective as an adjunct to opioids in treating neuropathic pain 

Smoking cannabis has been shown to decrease self-reported pain scores in men with complex regional

pain syndrome

Page 136: Pain Managment

Answer

• Cannabinoids such as dronabinol are no more effective than codeine

Page 137: Pain Managment

A qualitative systematic review in the British Medical Journal examined the safety and

efficacy of cannabinoids for treatment of pain. It showed that cannabinoid doses equivalent to 5-20 mg of 9-THC were no more effective than 50-120 mg of codeine, and some were no

better than placebo (evidence level I). Codeine is the least effective opioid agent. In

this study, cannabinoids were commonly associated with central nervous system

depression and undesirable psychotropic effects. Their usefulness for pain control is

therefore quite limited.

Page 138: Pain Managment

True statements regarding the management of chronic pelvic pain include which of the following?

(Mark all that are true.)Addressing patients' social issues may be helpful in

resolving symptomsChronic pelvic pain patients should be managed by a

gynecologist or pain specialistWhen chronic pelvic pain is cyclic, diagnostic

laparoscopy is required before starting hormonal therapy

When chronic pelvic pain is associated with gastrointestinal symptoms, a trial of diet

modification and/or antispasmodics is appropriateA complete evaluation will usually uncover a

specific cause of most chronic pelvic pain

Page 139: Pain Managment

Answer

• Addressing patients' social issues may be helpful in resolving symptomsWhen chronic pelvic pain is associated with gastrointestinal symptoms, a trial of diet modification and/or antispasmodics is appropriate

Page 140: Pain Managment

Addressing psychological and social issues that commonly occur in association with chronic pelvic pain may be important in resolving symptoms (B level

recommendation). Depression and sleep disorders are common in women with chronic pain. This may be a consequence rather than a cause of their pain, but

specific treatment may improve the woman's ability to function (Evidence level III). The multifactorial nature of chronic pelvic pain should be discussed and explored from the start. The aim should be to develop a partnership between

clinician and patient to plan a management program (A level recommendation).

Many women with chronic pelvic pain can be managed by their primary care physician. Family physicians might consider referral when the pain has not been explained to the woman's satisfaction or when pain is inadequately controlled.

Women with cyclical pain should be offered a therapeutic trial using the combined oral contraceptive pill or a GnRH agonist for a period of 3-6 months before having diagnostic laparoscopy. The levonorgestrel-releasing intrauterine

system could also be considered (A level recommendation).

Women with associated gastrointestinal symptoms should be offered a trial of antispasmodics (A level recommendation). They should also try amending their

diet to control symptoms (B level recommendation).

Page 141: Pain Managment

Patients experiencing acute pain due to sickle cell crisis often obtain poor relief of their pain. Factors that contribute to this lack of

effective relief include which of the following?

(Mark all that are true.)Lack of frequent reassessment of the

effectiveness of treatmentConcerns about addiction

Ethnic minority statusConfusion between addiction, tolerance, and

physical dependenceLack of effective medicines for this type of

pain

Page 142: Pain Managment

Answer

• Lack of frequent reassessment of the effectiveness of treatmentConcerns about addictionEthnic minority statusConfusion between addiction, tolerance, and physical dependence

Page 143: Pain Managment

Sickle cell crisis can produce severe pain, and may require the use of oral, intravenous, or patient-

controlled analgesia. Many of the reasons for poor relief of pain during sickle cell crisis are the same as

those for poor relief of acute pain in general: misconceptions and exaggerated concerns about

addiction, low priority given to pain relief, and lack of appropriate assessment and reassessment. In

addition, ethnic minority status has been shown to play a role in many health care disparities, including

pain management. Opioids are very effective in relieving the acute pain of sickle cell crisis (A level

recommendation).

Page 144: Pain Managment

A 64-year-old female has been receiving hospice services for 4 months because of metastatic breast cancer. She has had right chest wall pain since her right mastectomy 6 years ago. Since she began hospice care, this has been well

controlled on sustained-release morphine, 30 mg every 12 hours, with rare use of 15 mg every 2 hours as needed.

The hospice nurse calls you on a Monday morning to report that the patient's pain has become much worse over the previous 24 hours. The patient has been taking

her immediate-release morphine every 2 hours for 18 hours, but still rates the pain as 8 on a scale of 0-10, and has not slept all night.

Which of the following actions would be appropriate choices for managing this acute pain?

(Mark all that are true.)Double the dose of immediate-release morphine and have the nurse call back after

90 minutes to report the effectChange to a different opioid, beginning with 50% of the equianalgesic dose

Add amitriptyline, 25 mg at bedtime, as an adjuvant analgesic and to help her sleep better

Have the hospice nurse give 10 mg morphine subcutaneously now, then repeat in 20 minutes if the patient still rates the pain as 5 or greater

Page 145: Pain Managment

Answer

• Double the dose of immediate-release morphine and have the nurse call back after 90 minutes to report the effectHave the hospice nurse give 10 mg morphine subcutaneously now, then repeat in 20 minutes if the patient still rates the pain as 5 or greater

Page 146: Pain Managment

This patient is experiencing a pain crisis of rapid onset. Severe pain is a medical emergency and should be treated as such with aggressive titration

of opioids: a 50%-100% increase in dose is appropriate for initial treatment, either orally or parenterally. The effect should be assessed

within 90 minutes for oral administration or 15-20 minutes for subcutaneous administration.

Tolerance to opioids develops gradually, and the patient's history of abrupt worsening over 24 hours suggests that this does not explain her pain. Tricyclic antidepressants may be helpful in the management of

chronic pain, but do not typically have a role in the management of a pain crisis.

Opioid rotation is sometimes helpful if pain is refractory to aggressive titration, but should not be an initial approach to a patient with a pain

crisis.

In addition to rapid treatment of the severe pain, assessment of its etiology is vital. Appropriate assessment of the sudden onset of severe

chest pain in this hospice patient includes, at a minimum, inquiries about other related symptoms and physical findings. Further evaluation may be

needed for optimal management.

Page 147: Pain Managment

A 59-year-old female has persistent pain months after an acute herpes zoster outbreak in a right T-10

distribution. There are no active lesions and only mild erythema remains where the rash appeared. Her

pain is localized to the back. It is persistent and severe and interferes with her daily routines.

Which of the following treatment courses would be good choices?

(Mark all that are true.)Opioid analgesia

Tricyclic antidepressantsCorticosteroidsAnticonvulsants

Nerve blocks

Page 148: Pain Managment

Answer

• Opioid analgesiaTricyclic antidepressantsAnticonvulsants

• Nerve blocks

Page 149: Pain Managment

Postherpetic neuralgia can be a severe continuing problem. Opioids can have a role in the treatment of

acute herpes zoster outbreaks, but in postherpetic neuralgia should be limited to control of extreme

pain when initiating a treatment plan. Antidepressant medications, particularly tricyclics, can be very

helpful (Evidence level I). In addition to their role in pain control, they may be helpful in treating the

depression that can be seen in almost 90% of patients with postherpetic neuralgia. Corticosteroids

are not useful in postherpetic neuralgia. Anticonvulsants such as gabapentin are helpful

(Evidence level I). Nerve blocks have been successful in achieving pain relief, as have

transdermal anesthetics.

Page 150: Pain Managment

True statements regarding tricyclic antidepressants include which of the following? (Mark all that are true.)

Analgesic efficacy and side effects are both dose relatedBecause of the anticholinergic side effects, tertiary amines such as amitriptyline should be avoided in elderly patients

Amitriptyline has demonstrated benefits for acute pain, neuropathic pain, fibromyalgia, and low back pain

While tricyclic antidepressants can exacerbate existing cardiac conduction abnormalities, this problem does not arise

with the doses typically used for pain managementThe onset of effect for pain relief is similar to that required

for an antidepressant effect

Page 151: Pain Managment

Answer

• Analgesic efficacy and side effects are both dose relatedBecause of the anticholinergic side effects, tertiary amines such as amitriptyline should be avoided in elderly patientsAmitriptyline has demonstrated benefits for acute pain, neuropathic pain, fibromyalgia, and low back pain

Page 152: Pain Managment

Tricyclic antidepressants (TCAs) have been studied in a number of controlled trials and have demonstrated benefits as analgesics (evidence

level I). Amitriptyline has been studied most thoroughly and is efficacious in a number of pain states, including acute pain, neuropathic pain,

fibromylagia, and low back pain. Amitriptyline is a tertiary amine and has significant anticholinergic side effects, including dry mouth, constipation,

urinary retention, sedation, and weight gain. For these reasons, amitriptyline should be avoided in the elderly (C recommendation). The secondary amines (desipramine, nortriptyline) have less anticholinergic

activity and may be better tolerated.

Cardiac conduction abnormalities, recent cardiac events, and narrow-angle glaucoma are contraindications to using tricyclic antidepressants.

Both analgesic efficacy and side effects are dose dependent, with analgesic efficacy occurring at doses lower than those required to treat depression. Remarkably, analgesia from TCAs occurs in the absence of

depression or in cases with no antidepressant effect. The onset of analgesia occurs within 1 week, compared to the 3 weeks required for an

antidepressant effect.

Page 153: Pain Managment

A 45-year-old male is evaluated for a chronic cough and chest CT shows a suspicious lesion. An open lung biopsy is planned. The patient has chronic low back pain and takes

methadone, 10 mg orally 3 times a day. He has been on this stable dose for several years.

You meet with the patient and anesthesia team to plan for postoperative pain management. True statements regarding

this situation include which of the following? (Mark all that are true.)

The usual dose of methadone should be continued before and on the day of surgery, and restarted postoperatively

Alternative analgesia (e.g., epidural) should be used in the postoperative period while he is NPO

For moderate to severe pain, patient-controlled analgesia and epidural analgesia have a similar efficacy

The failure rate for epidural analgesia exceeds 15%Compared to NSAIDs, acetaminophen provides comparable

analgesia for postoperative pain

Page 154: Pain Managment

Answer• The usual dose of methadone should be

continued before and on the day of surgery, and restarted postoperativelyAlternative analgesia (e.g., epidural) should be used in the postoperative period while he is NPOThe failure rate for epidural analgesia exceeds 15%Compared to NSAIDs, acetaminophen provides comparable analgesia for postoperative pain

Page 155: Pain Managment

Methadone is a synthetic long-acting opioid used for chronic pain management and treating opioid addiction. All patients on methadone, whether for methadone maintenance treatment or pain management, should continue the dose before and

on the day of the surgery to avoid unnecessary fluctuation of the drug level (C level recommendation). The practice of abrupt discontinuation of methadone

before surgery is unjustifiable. Patients should resume oral methadone as soon as they can tolerate oral fluids well. During the period of fasting in the postoperative period, patients should receive alternative analgesia such as intravenous patient-controlled analgesia or regional analgesia/anesthesia (C level recommendation).

While patient-controlled analgesia (PCA) and epidural analgesia are commonly used for postoperative pain control, epidural analgesia is generally considered

more effective. There is a lower incidence of moderate to severe pain and severe pain when an epidural is used (20.9% and 7.8% respectively) compared with PCA

(35.8% and 10.4%) (Evidence level I). Large prospective studies of epidural analgesia report a 17.4% analgesic failure rate (e.g., catheter dislodgement,

unilateral block, missed segment).

In major surgery, the efficacy of NSAIDs and acetaminophen seems to be comparable (Evidence level I). Acetaminophen is a viable alternative to NSAIDs,

especially because of the low incidence of adverse effects, and should be the preferred choice in high-risk patients (C level recommendation).

Page 156: Pain Managment

You have been treating one of your colleagues for chronic pain over the past several months and you now suspect addiction problems. True statements regarding impaired

physicians include which of the following? (Mark all that are true.)

Chemically dependent physicians are more difficult to identify and treat than regular patients

As many as 10% of physicians abuse alcohol, opioids, sedatives, or stimulants

When addiction begins to affect coworkers or otherwise becomes evident in the office or hospital setting, it is a

longstanding problemPhysicians have an ethical obligation to report impaired

colleaguesThe physician providing treatment is best suited to monitor

the impaired physician's fitness to work

Page 157: Pain Managment

Answer

• Chemically dependent physicians are more difficult to identify and treat than regular patientsAs many as 10% of physicians abuse alcohol, opioids, sedatives, or stimulantsWhen addiction begins to affect coworkers or otherwise becomes evident in the office or hospital setting, it is a longstanding problemPhysicians have an ethical obligation to report impaired colleagues

Page 158: Pain Managment

Physicians who are impaired for any reason must refrain from assuming patient responsibilities that they cannot discharge safely and effectively. Whenever there is doubt, they should seek assistance in caring

for their patients. 

Every physician is responsible for protecting patients from an impaired physician and for assisting an impaired colleague. Fear of being wrong, embarrassment, or possible litigation should not deter or delay identification of an impaired colleague. The identifying physician may find it helpful to discuss the issue

with the department chair or a senior member of the staff or community. 

Impairment may result from use of habit-forming agents (alcohol or other substances) or from psychiatric, physiologic, or behavioral disorders. Impairment may also be caused by diseases that affect the cognitive or motor skills necessary to provide adequate care. The presence of these disorders, or the

fact that a physician is being treated for them, does not necessarily imply impairment. 

Although the legal responsibility to do so varies among states, there is a clear ethical responsibility to report a physician who seems to be impaired to an appropriate authority (such as a chief of service, chief of staff, institutional committee, state medical board, or regulatory agency). Physicians should aid their impaired colleagues in identifying appropriate sources of help. While undergoing therapy, the impaired

physician is entitled to full confidentiality, as in any other patient-physician relationship. To protect patients of the impaired physician, someone other than the treating physician must monitor the impaired physician's fitness to work. Serious conflicts may occur if the treating physician tries to fill both roles.

A study by Hughes and colleagues found that 11.4% of physicians had used benzodiazepines in the previous year in an unsupervised fashion, and 17.6% had engaged in unsupervised use of opioids (Evidence level II). Clearly, this results in part from the enhanced access physicians have to these

substances as compared to the general population. At a minimum, these data suggest that a substantial minority of physicians are using these medications in what would often be considered a clinically

inappropriate fashion. In contrast, this study showed that physicians were less likely than the general population to have used tobacco and a variety of illicit substances, including marijuana, cocaine, and heroin, than community controls. Compared with controls, physicians are five times as likely to take

sedatives and minor tranquilizers without medical supervision.

Page 159: Pain Managment

For which of the following conditions do epidural corticosteroid injections have moderate to strong

evidence of efficacy? (Mark all that are true.)Cervical radiculopathyLumbar radiculopathy

Neck painLow back painSpinal stenosis

Page 160: Pain Managment

Answer

• Cervical radiculopathyLumbar radiculopathy

Page 161: Pain Managment

Epidural corticosteroid injections can be administered using interlaminar, transforaminal, and caudal approaches. While

they are commonly performed by pain specialists, the evidence of efficacy for these procedures is lacking. Multiple systematic reviews have provided conflicting opinions. There

is moderate evidence for short-term and long-term relief using interlaminer epidural injections for cervical

radiculopathy (evidence level 2). The evidence for epidural injections in managing lumbar radiculopathy was strong for short-term relief and limited for long-term relief (evidence

level 2). The evidence is inconclusive in the management of neck pain, low back pain, and lumbar spinal stenosis

(Evidence level C). The evidence is strong for short-term improvement and moderate for long-term improvement in the

management of nerve root pain.

Page 162: Pain Managment

The Federation of State Medical Boards has published guidelines for the treatment of pain with

controlled substances. These guidelines recommend which of the following? (Mark all that are true.

)Documentation of a complete history and physical examination

Documentation of the patient's treatment plan, including ways to measure treatment response

An accurate record of the medications prescribed and a follow-up plan

Having the patient return at appropriate intervals for reevaluation

A contract or written agreement between the patient and the physician

Page 163: Pain Managment

Answer

• Documentation of a complete history and physical examinationDocumentation of the patient's treatment plan, including ways to measure treatment responseAn accurate record of the medications prescribed and a follow-up planHaving the patient return at appropriate intervals for reevaluation

Page 164: Pain Managment

A contract or written agreement is not required by the Federation of State Medical Boards (FSMB) but should be considered if the patient is at high risk for

medication abuse. A contract typically includes medication dosages, the frequency of visits, and information about refills of prescriptions. The

guidelines do require a documented treatment plan and ways to measure treatment response should be

included. Also required are an accurate record of the medications and a follow-up plan. Central to a good record, and an FSMB requirement, is documentation of a complete history and physical examination. The FSMB and other pain experts believe that the patient should be seen at appropriate intervals to reevaluate

the treatment plan and assess the patient (C level recommendation for all recommendations).

Page 165: Pain Managment

True statements regarding chronic pain include which of the following?

(Mark all that are true.)It persists after the initial injury has

healedIt can be associated with hyperalgesia

It can be associated with allodyniaIt can spread to non-injured areas

It is associated with structural changes in the central nervous system

Page 166: Pain Managment

Answer

• It persists after the initial injury has healedIt can be associated with hyperalgesiaIt can be associated with allodyniaIt can spread to non-injured areasIt is associated with structural changes in the central nervous system

Page 167: Pain Managment

Patients and their physicians are familiar with acute pain or pain caused by injury. Injury leads to inflammation and changes within the central nervous system. Pain signals are sent to the brain. The brain in turn signals the muscles, causing a reflex muscle spasm. These changes protect the injured area. The tightening of the muscles forms a natural cast around the injury, and the negative sensation of pain promotes learning how to avoid similar injury in the future. As tissues heal, inflammation resolves and the central nervous system sends out fewer signals, resulting in decreased pain and decreased muscle spasm. 

Less is known about the etiology of chronic pain. Chronic pain often occurs in the absence of ongoing illness or after healing is complete, and often begins with an injury that causes inflammation and central nervous system changes. The

injured area heals, scar tissue is produced, and the inflammation resolves. But, for an unknown reason, the nervous system continues to send pain signals to somatic muscles, as though a new injury were occurring. The nervous system reacts to the

memory of the original injury and sends signals similar to those sent in response to that injury. These signals become a disabling message, reminding the patient of the injury (C level recommendation).

Hyperalgesia results when second-order neurons at the level of the dorsal horn become more sensitive to peripheral stimuli. They demonstrate increased numbers of action potentials and spontaneous discharges in response to painful stimuli. This

increased number of action potentials is experienced as an elevated response to painful stimuli that were previously perceived as less painful (C level recommendation).

Allodynia is the perception of pain caused by usually nonpainful stimuli, such as touch or vibration. Allodynia results from a redistribution of central terminals. Mechanoreceptors establish new synapses with dorsal horn cells that normally receive

nociceptive input. After redistribution, mechanoreceptors stimulated by touch or vibration will activate pain pathways in the same way they are activated by nociceptive neurons in response to pain (C level recommendation).

The spread of pain occurs because of an increase in the size of receptive fields within the dorsal horn. Pain perception then spreads to involve areas that are not normally innervated by the injured nerve.

Nerve injury may result in multiple changes within the central nervous system that perpetuate the pain experience. Increased numbers of action potentials cause hypersensitivity to pain. Redistribution of synapses for mechanoreceptors causes

allodynia. Increased receptive field size results in the spread of pain. The use of exercise and psychologic treatment may be effective in chronic pain because these treatments retrain the nervous system to reestablish more normal neural connections

(C level recommendation).

Page 168: Pain Managment

True statements regarding opioid-induced respiratory depression include which of the following?

(Mark all that are true.)It generally occurs in opioid-naive

patientsPain reduces this effect

It is rare when opioids are appropriately titrated

It occurs commonly in patients with COPD

Page 169: Pain Managment

Answer

• It generally occurs in opioid-naive patientsPain reduces this effectIt is rare when opioids are appropriately titrated

Page 170: Pain Managment

Respiratory depression generally occurs in opioid-naive patients given excessive initial doses (evidence

level III). It is rare when opioids are appropriately titrated, even when aggressive rapid titration is

required in the face of severe pain. This side effect of opioids is reduced by pain, and sudden relief of pain

(such as successful nerve block or a dramatic response to an adjuvant analgesic) may require rapid

downward titration of the opioid in order to avert respiratory depression. Patients with underlying

respiratory disease, including COPD, may be more vulnerable to respiratory depression, but even in these patients respiratory depression remains rare

when appropriate titration is used.

Page 171: Pain Managment

A 52-year-old male presents to your office with severe, constant abdominal pain that has kept him home from work for the last 2 days. When asked about the pain, he states that he has had similar abdominal pain before, but this time the

pain is much more intense. He is a recovering alcoholic who has been previously diagnosed with chronic pancreatitis.

True statements regarding pain management in this situation include which of the following?

(Mark all that are true.)Opioids cannot be used if a history of alcohol abuse existsA daily patient pain log is a good way to track chronic pain

Surgery is the definitive treatment for this diseasePancreatic enzymes coupled with H2-blockers are a good first

treatment

Page 172: Pain Managment

Answer

• A daily patient pain log is a good way to track chronic painPancreatic enzymes coupled with H2-blockers are a good first treatment

Page 173: Pain Managment

Painful chronic pancreatitis is poorly understood, and its management is controversial. Patients present with a wide variety of pain, ranging from mild to severe and constant to intermittent. The nature of chronic pancreatic pain only

confounds the response to treatment.

As with any chronic pain situation, it can be difficult to choose the appropriate pain therapy. This situation is compounded by a history of past or present alcohol

abuse/addiction. Opioids are not ruled out for pain management in alcohol abusers, but they do need to be selected with an awareness of their addictive

properties. In order to make a more objective decision, it is crucial to assess the nature, frequency, severity, and activity impact of the pain. A daily patient pain

log is a good way to reach this goal. 

Presently, there is no clear evidence that endoscopic or other surgical therapies for pain in chronic pancreatitis are beneficial. While there have been some promising results, there have been no controlled trials comparing surgery with either medical or endoscopic treatment. Further evaluation in clinical trials is needed to define a

good treatment plan.

Before the continuous use of opioids or any surgery is planned, the patient should be put on a trial of high-dose pancreatic enzymes coupled with H2-blockers (A level recommendation). This provides a good initial treatment plan that can be

adjusted later.

Page 174: Pain Managment

Which one of the following is true regarding management of osteoarthritis of the knee?

 When prescribing an exercise program, range-of-motion exercises should be included  

Tri-compartmental knee arthritis requires surgical treatment 

Good results have been reported for total knee arthroplasty in patients over 60 years of age 

Arthroscopic lavage should be considered even for patients without mechanical

symptoms

Page 175: Pain Managment

Answer

• When prescribing an exercise program, range-of-motion exercises should be included 

Page 176: Pain Managment

Nonpharmacologic osteoarthritis treatment modalities are directed toward weight reduction, joint protection, and energy conservation. The exercise program should include range-of-

motion (B level recommendation), aerobic, and muscle strengthening exercises. Patients may also require physical therapy, occupational therapy, assistive devices for ambulation and activities of daily living, and advice regarding appropriate footwear and orthotics (e.g.,

wedged insoles).

For pharmacologic therapy, the initial drug of choice is acetaminophen, 4 g/day. For patients taking NSAIDs, gastrointestinal (GI) risk should be assessed, including any history of ulcer

disease and/or GI bleeding. Other risk factors include the use of high-dose, chronic, or multiple NSAIDs, including aspirin; concomitant use of corticosteroids and/or warfarin (A

level recommendation); and age >60 years. A gastroprotective agent should be prescribed for patients determined to be at high risk.

Arthroscopic debridement may be indicated for the treatment of patients with degenerative arthritis with mechanical symptoms, such as locking, catching, or giving way of the joint (B level recommendation). Neither arthroscopic lavage nor debridement is indicated for patients

without mechanical symptoms (A level recommendation). Results of arthroscopic debridement in patients with mechanical symptoms are variable, but high success rates are

reported if there is no gross malalignment or instability, some articular cartilage remains, and symptoms are well localized (B level recommendation). 

Patients with bi- or tricompartmental arthritis of the knee should be considered for total knee arthroplasty only if conservative treatment is unsuccessful (A level recommendation). Good results have been reported in total knee arthroplasty only in patients under 55 years of age (A

level recommendation).

Page 177: Pain Managment

A 48-year-old male who works on the loading dock at a large distribution center strained his back 2 days ago while pushing a heavy dolly. When

you see him in your office, he walks haltingly into the examination room complaining of low back pain. His reflexes and sensation are normal and a straight leg raising test is negative. His muscle effort is compromised by

pain. He has spasm and tenderness and reduced range of motion in his lower back.

True statements regarding this situation include which of the following? (Mark all that are true.)

Close to 90% of patients with low back pain improve within 6 weeks of onset

One to two days of rest followed by a gradual return to activity gives the best functional results

All patients over age 45 with low back pain should have lumbar spine films

Uncontrolled pain is an indication for advanced imaging studiesAny loss of bowel or bladder control requires emergent evaluation

Page 178: Pain Managment

Answer

• Close to 90% of patients with low back pain improve within 6 weeks of onsetUncontrolled pain is an indication for advanced imaging studiesAny loss of bowel or bladder control requires emergent evaluation

Page 179: Pain Managment

Based on the history and physical examination, low back pain can be classified by duration and location. Low back pain can be categorized as acute or chronic low back pain, or acute

or chronic sciatica.

Approximately 70% or more of patients with low back pain will improve within 2 weeks, and 90% will improve within 6 weeks. Back pain that persists for longer than 6 weeks is classified as chronic. Among patients with chronic pain, 90% have low back pain and the

remainder have sciatica.

Management should emphasize patient education and conservative home self-care that includes bed rest, early ambulation, postural advice, gentle stretching, use of ice and heat,

nonprescription anti-inflammatory agents and analgesics, and early return to work or activity. Patients with acute back pain should be advised to stay active, as permitted by the pain, and patients with chronic pain should be advised that exercise is effective therapy (C

level recommendation).

A small percentage of patients with back pain have a serious underlying disorder, and these should be taken into consideration during the workup. These disorders include infection, malignancy, rheumatologic disease, neurologic disease, and problems that can cause pain

referred to the lower back. Patients with signs or symptoms of serious underlying problems should be seen within 24 hours. Spine radiographs should be performed only if there is a red

flag indicating one of these problems (Evidence level III).Advanced imaging studies are infrequently indicated. One indication is cauda equina

syndrome, which may be manifested by a sudden loss or change in bowel or bladder control or function.